Potpourri UWorld Flashcards

1
Q

5 factors that affect molecular rate of diffusion across a semipermeable membrane

A

Inversely proportional to molecular weight and membrane thickness. Proportional to particle concentration, surface area and solubility of the substance.

How well did you know this?
1
Not at all
2
3
4
5
Perfectly
2
Q

A patient presents with fever, lymphadenopathy, a skin rash and facial edema a few weeks after starting a new medication. What medications can cause this condition?

A

The patient has DRESS syndrome (drug reaction with eosinophilia and systemic symptoms). Common drugs that cause this are allopurinol, anticonvulsants, sulfonamides and antibiotics.

How well did you know this?
1
Not at all
2
3
4
5
Perfectly
3
Q

Target organs affected by alpha-1 stimulation?

A

Peripheral vasculature (vasoconstriction), bladder (contraction of internal urethral sphincter) and eye (mydriasis from pupillary dilator contraction)

How well did you know this?
1
Not at all
2
3
4
5
Perfectly
4
Q

Target organs affected by beta-1 stimulation?

A

Heart (increased contractility, chronotropy and conductance) and juxtaglomerular cells

How well did you know this?
1
Not at all
2
3
4
5
Perfectly
5
Q

Target organs affected by beta-2 stimulation?

A

Peripheral vasculature (vasodilation), bronchodilation and uterine relaxation.

How well did you know this?
1
Not at all
2
3
4
5
Perfectly
6
Q

Cytogenic results in a patient with fragile X syndrome.

A

X-linked mutation of the FMR1 gene results in CGG trinucleotide repeats. This results in cytogenic studies showing a gap at the end of the long arm of chromosome X because the CGG expanded region does not stain when cultured in folate deficient media.

How well did you know this?
1
Not at all
2
3
4
5
Perfectly
7
Q

A newborn child of a mother with IV drug use presents with tachypnea, crying, sneezing and diarrhea 2 days after birth. How do you treat this child?

A

The child has neonatal abstinence syndrome, best treated with opiate replacement and gradual weaning.

How well did you know this?
1
Not at all
2
3
4
5
Perfectly
8
Q

How does IgA protease help out bacteria like N. meningitides, N. gonorrhoeae, S. pneumo and H. influenzae?

A

IgA typically binds to capsular antigens like fimbriae that typically allow for bacterial adherence. IgA protease cleaves IgA and allows for increased adherence to mucosal surfaces.

How well did you know this?
1
Not at all
2
3
4
5
Perfectly
9
Q

A 7 year old boy presents with facial hair, paralysis of upward gaze and convergence. MRI shows obstructive hydrocephalus. What is likely causing his condition?

A

A beta-hCG-secreting pineal germinoma.

How well did you know this?
1
Not at all
2
3
4
5
Perfectly
10
Q

Lateral medullary syndrome

A

Contralateral loss of pain and temperature sensation with ipsilateral loss of CN V, IX, X and XI function.

How well did you know this?
1
Not at all
2
3
4
5
Perfectly
11
Q

Medial medullary syndrome

A

Contralateral spastic paralysis and ipsilateral flaccid paralysis of the tongue (CN XII)

How well did you know this?
1
Not at all
2
3
4
5
Perfectly
12
Q

Consequence of common peroneal nerve injury?

A

Foot drop due to paralysis of peroneus longus, peroneus brevus, tibialis anterior and extrinsic toe extensors. There is also numbness along the anterolateral leg and dorsal foot.

How well did you know this?
1
Not at all
2
3
4
5
Perfectly
13
Q

Consequence of superficial peroneal nerve injury?

A

Poor eversion due to paralysis of peroneus longus and peroneus brevis and loss of sensation along the distal anterolateral leg. However, dorsiflexion is preserved because the deep peroneal nerve innervates the tibialis anterior.

How well did you know this?
1
Not at all
2
3
4
5
Perfectly
14
Q

What diseases result in metabolic impairments in glycogenolysis?

A

Type I (von Gierke disease): deficiency in the conversion of Glc-6-P to Glc by Glc-6-Phosphatase. This results in fasting hypoglycemia, lactic acidosis, hyperuricemia, hyperlipidemia, STEATOSIS and hepatomegaly.

Type II (Pompe disease): deficiency in acid maltase, resulting in inability to degrade glycogen in lysosomes. This results in normal blood glucose, severe cardiomegaly and GLYCOGEN ACCUMULATION IN LYSOSOMES.

Type III (Cori disease): deficiency in debranching enzyme resulting in accumulation of abnormal glycogen with short outer chains (limit dextrins), WEAKNESS AND HYPOTONIA, ketotic hypoglycemia and hepatomegaly.

Type V (McArdle disease): deficiency in muscle phosphorylase resulting in inability to phosphorylate glycogen causing weakness with exercise and LOW POST-EXERCISE BLOOD LACTATE and MYOGLOBINURIA.

How well did you know this?
1
Not at all
2
3
4
5
Perfectly
15
Q

Mechanism of opioid-induced hyperalgesia

A

Chronic mu receptor stimulation causes increased turnover of inhibitory opioid receptors, decoupling of receptors from second messenger system and upregulation of NMDA receptors.

How well did you know this?
1
Not at all
2
3
4
5
Perfectly
16
Q

Type of orbital fracture that may impair corneal reflex

A

Those affecting the superior orbital fissure because CNs III, IV, V and VI pass through here and the corneal reflex involves V1 sensation and VII motor action.

How well did you know this?
1
Not at all
2
3
4
5
Perfectly
17
Q

Physical exam findings suggestive of an orbital floor fracture?

A

Limited superior gaze due to inferior rectors entrapment and numbness of the upper cheek, lip and gingiva due to damage to the infraorbital nerve that normally runs along the floor of the orbit (V2).

How well did you know this?
1
Not at all
2
3
4
5
Perfectly
18
Q

1st line treatment of patients with disorders involving the urea cycle?

A

Protein restrict so the patient gets the AAs they need while minimizing ammonia production.

How well did you know this?
1
Not at all
2
3
4
5
Perfectly
19
Q

A 17 year old baseball player presents with upper extremity numbness, tingling and weakness in his right hand. He also complains of pain in exertion only in his right arm and right arm swelling. What is most likely causing his condition?

A

Thoracic outlet syndrome. The thoracic outlet is the space behind the clavicle and above the first rib and compression of the brachial plexus, subclavian vein and subclavian artery can occur with anterior or middle scalene muscle abnormalities, anomalous cervical rib or repeat overhead injuries.

How well did you know this?
1
Not at all
2
3
4
5
Perfectly
20
Q

What test has PYR-positivity replaced

A

Bacitracin resistance test to determine S. pyogenes (sensitive) from S. agalactiae (resistant)

How well did you know this?
1
Not at all
2
3
4
5
Perfectly
21
Q

What is the microbial differential for patients with necrotizing fasciitis?

A

S. pyogenes, S. aureus and C. perfringens.

How well did you know this?
1
Not at all
2
3
4
5
Perfectly
22
Q

What 3 cell types are contained in the juxtaglomerular apparatus?

A

Macula densa: tall narrow cells in the distal tubule that monitor flow rate and [Na] and send signals to juxtaglomerular cells.

Juxtaglomerular cells: modified smooth muscle cells in the afferent arteriolar wall that secrete zymogen granules in response to macula densa stimulation.

Extra-glomerular mesangial cells

How well did you know this?
1
Not at all
2
3
4
5
Perfectly
23
Q

Lymph nodes that receive lymphatic drainage from testes

A

Para-aortic

How well did you know this?
1
Not at all
2
3
4
5
Perfectly
24
Q

Lymph nodes that receive lymphatic drainage from glans of penis and superficial inguinal nodes

A

Deep inguinal nodes

How well did you know this?
1
Not at all
2
3
4
5
Perfectly
25
Q

Lymph nodes that receive lymphatic drainage from the scrotum

A

Superficial inguinal nodes

How well did you know this?
1
Not at all
2
3
4
5
Perfectly
26
Q

An immunocompromised patient presents with sinusitis and fungal hyphae that branch at acute angles with septations.

A

Aspergillus

How well did you know this?
1
Not at all
2
3
4
5
Perfectly
27
Q

An immunocompromised patient presents with sinusitis and fungal hyphae that branch at wide angles without septations.

A

Rhizopus species -> mucormycoses

How well did you know this?
1
Not at all
2
3
4
5
Perfectly
28
Q

What is the main virulence factor involved in E. coli-related meningitis

A

K1 capsular polysaccharide. It prevents phagocytosis and complement-mediated lysis.

How well did you know this?
1
Not at all
2
3
4
5
Perfectly
29
Q

Mechanisms of LPS-induced shock

A

Widespread cytokine release of IL-1, IL-6 and TNF-alpha

How well did you know this?
1
Not at all
2
3
4
5
Perfectly
30
Q

Main virulence factor involved in E. coli related blood diarrhea

A

Verotoxin (shiva-like) -> 60S inactivation -> cell death due to decreased protein synthesis

How well did you know this?
1
Not at all
2
3
4
5
Perfectly
31
Q

Main virulence factor involved in E. coli related watery diarrhea

A

Heat stable/labile enterotoxins promote fluid and electrolyte secretion from intestinal epithelium.

How well did you know this?
1
Not at all
2
3
4
5
Perfectly
32
Q

Main virulence factor involved in E. coli related UTI

A

P. fimbria allow for adherence to urogenital epithelium

How well did you know this?
1
Not at all
2
3
4
5
Perfectly
33
Q

Gram-positive, catalase-negative, alpha-hemolytic, optichin-sensitive and bile soluble.

A

S. pneumoniae

How well did you know this?
1
Not at all
2
3
4
5
Perfectly
34
Q

Gram-positive, catalase-negative, alpha-hemolytic, optichin-resistant and bile insoluble

A

S. viridans

How well did you know this?
1
Not at all
2
3
4
5
Perfectly
35
Q

Gram-positive, catalase-negative, beta-hemolytic, PYR-positive and bacitracin sensitive.

A

S. pyogenes

How well did you know this?
1
Not at all
2
3
4
5
Perfectly
36
Q

Gram-positive, catalase-negative, beta-hemolytic PYR-negative and bacitracin resistant.

A

S. agalactiae (GBS)

How well did you know this?
1
Not at all
2
3
4
5
Perfectly
37
Q

Gram-positive, catalase-negative, gamma-hemolytic, PYR positive, grows in bile and 6.5% NaCl.

A

Enterococci

How well did you know this?
1
Not at all
2
3
4
5
Perfectly
38
Q

Gram-positive, catalase-negative, gamma-hemolytic, PYR negative, grows in bile but not in 6.5% NaCl.

A

S. bovis

How well did you know this?
1
Not at all
2
3
4
5
Perfectly
39
Q

Why do patients with phenylketonuria have a mousy odor and poor pigmentation?

A

Excess Phe inhibits synthesis of melanin, causing poor pigmentation. Accumulation of abnormal Phe metabolites contributes to the mousy odor.

How well did you know this?
1
Not at all
2
3
4
5
Perfectly
40
Q

Enzyme deficiency in maple syrup urine disease

A

Branched chain ketoacid dehydrogenase deficiency results in an inability to decarboxylate the alpha-ketoacid derivatives of Leu, Ile and Val.

How well did you know this?
1
Not at all
2
3
4
5
Perfectly
41
Q

Enzyme deficiency that may result in ptosis, orthostasis, hypoglycemia and hypothermia.

A

Dopamine hydroxylase deficiency results in poor conversion of dopamine to norepinephrine and dysautonomia.

How well did you know this?
1
Not at all
2
3
4
5
Perfectly
42
Q

Enzyme deficiency resulting in excess accumulation of homogentistic acid, connective tissue hyper pigmentation and degenerative joint disease

A

Homogentistic acid dioxygenase -> poor tyrosine degradation -> Alkaptonuria

How well did you know this?
1
Not at all
2
3
4
5
Perfectly
43
Q

Enzyme deficiency that commonly causes albinism

A

Tyrosinase inactivity leads to poor conversion of tyrosine to DOPA and dopaquinone in the synthesis of melanin.

How well did you know this?
1
Not at all
2
3
4
5
Perfectly
44
Q

Histopathology of gynecomastia

A

Ductal epithelial hyperplasia surrounded by dense stromal fibrosis.

How well did you know this?
1
Not at all
2
3
4
5
Perfectly
45
Q

Mechanisms of gynecomastia

A

Increased aromatase activity: obesity & cirrhosis
Increased LH secretion: hyperthyroidism
Decreased testosterone production: CKD, androgen deprivation therapy, spironolactone
Decreased estrogen clearance: cirrhosis

How well did you know this?
1
Not at all
2
3
4
5
Perfectly
46
Q

Hormonal sequence resulting from water deprivation

A

ADH release -> V2 stimulation on principal cells in collecting duct -> aquaporin-2 insertion into apical membrane -> increased H2O reabsorption at collecting duct

How well did you know this?
1
Not at all
2
3
4
5
Perfectly
47
Q

Region of the nephron with the lowest osmolarity

A

Distal convoluted tubule, it is relatively impermeable to water and immediately follows NaCl loss from the ascending loop of Henle. Osmolarity in this region is typically ~ 100.

How well did you know this?
1
Not at all
2
3
4
5
Perfectly
48
Q

G-protein coupled receptors associated with the different adrenergic receptors? Cholinergic receptors? Dopaminergic receptors?

A
Alpha-1 = Gq -> activation of phospholipase C -> DAG & IP3 formed from membrane degradation -> PKC activated by DAG, Ca release from sarcoplasmic reticulum from IP3
Alpha-2 = Gi -> inhibition of adenylyl cyclase
Beta-1 = Gs -> activation of adenylyl cyclase -> increased cAMP -> PKA
Beta-2 = Gs
M1 = Gq
M2 = Gi
M3 = Gq
DA1= Gs -> increased cAMP
DA2 = Gi
How well did you know this?
1
Not at all
2
3
4
5
Perfectly
49
Q

What infections can result in brown-pigmented gallstones?

A

Infections in the biliary tract that release beta-glucuronidase from injury of hepatocytes and bacterial secretion. The beta-glucuronidase hydrolyzes bili glucuronides and leads to elevated levels of unconjugated bilirubin. Infections that can do this include E. coli, ascaris and opisthorchis.

How well did you know this?
1
Not at all
2
3
4
5
Perfectly
50
Q

Most effective medication for reducing triglycerides

A

Fibrates. They activate PPAR-alpha -> increased lipoprotein lipase activity

How well did you know this?
1
Not at all
2
3
4
5
Perfectly
51
Q

How do corticosteroids affect the WBC?

A

Increased PMNs (mobilization of marginated pool) and reduction of lymphs (T > B reduced within minutes), monos, basos and eos.

How well did you know this?
1
Not at all
2
3
4
5
Perfectly
52
Q

Cause of ataxia-telangectaisa

A

AR mutation in the ATM gene, which is responsible for DNA breakdown and repair. Common manifestation of disease is recurrent pulmonary infections and IgA deficiency.

How well did you know this?
1
Not at all
2
3
4
5
Perfectly
53
Q

Cause of Niemann-Pick disease

A

AR mutation in sphingomyelinase that causes accumulation of sphingomyelin in monocytes leading to HSM, anemia, areflexia, a macular cherry red spot and early death in childhood.

How well did you know this?
1
Not at all
2
3
4
5
Perfectly
54
Q

When is beta-hCG first detectable in maternal serum?

A

8 days. Around days 4-5 the blastocyst forms. Day 6 it implants and differentiates into the cytotrophoblast and syncytiotrophoblast. The syncytiotrophoblast is what invades the endometrium and starts secreting beta-hCG, levels will rise to detectable levels by day 8.

How well did you know this?
1
Not at all
2
3
4
5
Perfectly
55
Q

Side effects to be aware of when prescribing a diabetic canagliflozin?

A

SGLT-2 inhibits increase urinary [glucose] significantly. Urinary mycotic infections and symptomatic hypotension from osmotic diuresis are the most significant side effects. SGLT-2 inhibitors are contraindicated in patients with moderate to severe renal insufficiency.

How well did you know this?
1
Not at all
2
3
4
5
Perfectly
56
Q

How do the insulin secretagogues work?

A

Sulfonylureas and metaglinides inhibit the K+ ATP channels in beta cells to increase insulin secretion.

How well did you know this?
1
Not at all
2
3
4
5
Perfectly
57
Q

How do the biguanides work?

A

They simulate AMPK activity and reduce peripheral tolerance and glucose production.

How well did you know this?
1
Not at all
2
3
4
5
Perfectly
58
Q

Mechanism of and side effects to watch out for with pioglitazone.

A

Thiazolidinediones work by activating PPAR-gamma and reduce insulin resistance, but can cause worsening of heart failure due to fluid retention.

How well did you know this?
1
Not at all
2
3
4
5
Perfectly
59
Q

Mechanism of and side effects to watch out for with exenatide and liraglutide.

A

GLP-1 agonists delay gastric emptying, increase glucose-dependent insulin secretion and decrease glucagon secretion. Look out for pancreatitis.

How well did you know this?
1
Not at all
2
3
4
5
Perfectly
60
Q

Mechanism of and side effects to watch out for with sitagliptin and saxagliptin.

A

DPP4 inhibitors increase endogenous GLP and GIP levels. Look out for nasopharyngitis.

How well did you know this?
1
Not at all
2
3
4
5
Perfectly
61
Q

Mechanism of and side effects to watch out for with acarbose and miglitol?

A

Alpha-glucosidase inhibitors reduce intestinal absorption of carbohydrates. Look out for diarrhea and flatulence.

How well did you know this?
1
Not at all
2
3
4
5
Perfectly
62
Q

Endoderm gives rise to…

A

Respiratory epithelium, biliary tree, pancreas, GI epithelium and bladder epithelium.

How well did you know this?
1
Not at all
2
3
4
5
Perfectly
63
Q

Mesoderm gives rise to…

A

Dermis, bone, muscle, blood vessels and most visceral tissue.

How well did you know this?
1
Not at all
2
3
4
5
Perfectly
64
Q

Neurectoderm gives rise to…

A

CNS, pre-ganglionic autonomic neurons, retina and posterior pituitary.

How well did you know this?
1
Not at all
2
3
4
5
Perfectly
65
Q

Ectoderm gives rise to…

A

Skin, lens of the eye, anterior pituitary and mammary glands.

How well did you know this?
1
Not at all
2
3
4
5
Perfectly
66
Q

Stains used to diagnose cryptococcal meningitis

A

Mucicarmine and methenamine silver staining

How well did you know this?
1
Not at all
2
3
4
5
Perfectly
67
Q

Where on IgG do antigen and complement bind?

A

Antigen binds to the Fab “Y” region. C1 binds to Fc region near the hinge point after Ig binds antigen.

How well did you know this?
1
Not at all
2
3
4
5
Perfectly
68
Q

Why is IgM so much better at activating complement than IgG if they have the same complement binding site?

A

IgM circulates in its pentameric form and IgG circulates in its monomeric form.

How well did you know this?
1
Not at all
2
3
4
5
Perfectly
69
Q

Renal plasma flow calculation? How do you get the renal blood flow calculation?

A

Renal plasma flow = (urine[PAH] x urine flow rate) / plasma[PAH]

Renal blood flow = renal plasma flow / (1 - Hct)

How well did you know this?
1
Not at all
2
3
4
5
Perfectly
70
Q

What is the filtration fraction

A

GFR:Renal plasma flow. i.e. the amount of plasma filtered by the glomerulus relative to the amount of plasma that comes through the capillaries.

How well did you know this?
1
Not at all
2
3
4
5
Perfectly
71
Q

ACL attachment on the tibia

A

Anterior intercondylar fossa

How well did you know this?
1
Not at all
2
3
4
5
Perfectly
72
Q

Biceps femoris insertion site

A

Styloid process of head of the fibula

How well did you know this?
1
Not at all
2
3
4
5
Perfectly
73
Q

5 types of DCIS

A
Comedocarcinoma: solid sheets of highly pleomorphic cells with central necrosis.
Solid
Cribriform
Papillary
Micro papillary
How well did you know this?
1
Not at all
2
3
4
5
Perfectly
74
Q

p53 gene function

A

Tumor suppressor gene that causes G1 arrest if there are molecular abnormalities detected in the genome. If the damage is irreparable, p53 induces apoptosis

How well did you know this?
1
Not at all
2
3
4
5
Perfectly
75
Q

Result of chromosome 9;22 translocation

A

Bcr-abl fusion results in constitutively active tyrosine kinase, cellular proliferation and inhibition of apoptosis resulting in CML.

How well did you know this?
1
Not at all
2
3
4
5
Perfectly
76
Q

Ras gene function

A

Ras is a proto-oncogene in the MAP-kinase pathway that increases cellular sensitivity to mitogenic stimuli.

How well did you know this?
1
Not at all
2
3
4
5
Perfectly
77
Q

Obturator innervations

A

MOTOR: obturator externus, the posterior branches innervate the thigh adductors (magnus, longus and brevis)
SENSORY: the anterior division provides sensation for the distal medial thigh

How well did you know this?
1
Not at all
2
3
4
5
Perfectly
78
Q

Nerve that mediates thigh abduction? Thigh extension?

A

Superior gluteal nerve, it innervates gluteus medius, gluteus minimus and tensor fascia lata. Thigh extension = inferior gluteal nerve’s innervation of gluteus maximus.

How well did you know this?
1
Not at all
2
3
4
5
Perfectly
79
Q

Most common subtype of primary CNS lymphoma in patients who are immunocompromised

A

Diffuse large B-cell lymphoma, CD20+ and CD79a+

How well did you know this?
1
Not at all
2
3
4
5
Perfectly
80
Q

Mutations that increase the risk of developing a pheochromocytoma

A

VHL, RET and NF1

How well did you know this?
1
Not at all
2
3
4
5
Perfectly
81
Q

Pheochromocytoma rule of 10’s

A

10% extra-adrenal (paraganglioma), 10% bilateral and 10% malignant

How well did you know this?
1
Not at all
2
3
4
5
Perfectly
82
Q

How do oral contraceptives minimize hirsutism in patients with PCOS?

A

They suppress LH release from the pituitary, decreasing ovarian androgen production. They also increase SHBG production by the liver.

How well did you know this?
1
Not at all
2
3
4
5
Perfectly
83
Q

Why does H+ increase O2 dissociation from hemoglobin?

A

They hang around the histidine residues on Hgb, stabilizing Hgb in the taut state and facilitating unloading of O2 (Bohr effect)

How well did you know this?
1
Not at all
2
3
4
5
Perfectly
84
Q

Haldane effect

A

Increasing pO2 results in unloading of CO2 and H+.

How well did you know this?
1
Not at all
2
3
4
5
Perfectly
85
Q

Hawthorne effect

A

Changing one’s behavior as a result of knowing you are being observed

How well did you know this?
1
Not at all
2
3
4
5
Perfectly
86
Q

Post-strep GN cause of glomerular injury and biopsy characteristics.

A

Anti-strep abs cross react with glomerular basement membrane.
IF = granular C3 staining along GBM
EM = subEPIthelial humps

How well did you know this?
1
Not at all
2
3
4
5
Perfectly
87
Q

Anti-GBM cause of glomerular injury and biopsy characteristics

A

Anti-type IV collagen abs attack GBM.
IF = linear IgG and C3 staining along GBM
LM = glomerular crescents

How well did you know this?
1
Not at all
2
3
4
5
Perfectly
88
Q

Rapidly progressive glomerulonephritis cause of glomerular injury and biopsy characteristics

A

Severe immunologic injury (anti-GBM, immune complex depositions)
IF = fibrinogen in crescents
LM = glomerular crescents

How well did you know this?
1
Not at all
2
3
4
5
Perfectly
89
Q

IgA glomerulonephropathy cause of glomerular injury and biopsy characteristics

A

IgA complex deposition
IF = IgA in mesangium
LM = mesangial hypercellularity

How well did you know this?
1
Not at all
2
3
4
5
Perfectly
90
Q

Alport syndrome cause of glomerular injury and biopsy characteristics

A

Defective congenital type IV collagen -> BM splitting

EM = lamellated appearance of GBM

How well did you know this?
1
Not at all
2
3
4
5
Perfectly
91
Q

Biopsy characteristics of membranoproliferative glomerulonephritis

A
LM = lobular glomeruli with increased mesangial matrix, BM splitting with silver or PAS stain
IF = granular deposits
How well did you know this?
1
Not at all
2
3
4
5
Perfectly
92
Q

Focal segmental glomerulosclerosis biopsy results

A

IgM and C3 deposits in sclerotic regions of glomerulus

How well did you know this?
1
Not at all
2
3
4
5
Perfectly
93
Q

Enzyme needed to break down lactose into glucose and galactose.

A

The intestinal brush border enzyme disaccharidase

How well did you know this?
1
Not at all
2
3
4
5
Perfectly
94
Q

Drug used for treatment of wet macular degeneration?

A

Ranibizumab or bevacizumab (VEGF inhibitors). Patients with dry MD should receive zinc as an antioxidant to prevent drussen accumulation, ischemia, increased VEGF release and progression to wet MD.

How well did you know this?
1
Not at all
2
3
4
5
Perfectly
95
Q

Speed of hemoglobin movement in gel electrophoresis

A

HgbA (most negatively charged) > HgbS > HgbC (least negatively charged due to lysine substitution for glutamate)

How well did you know this?
1
Not at all
2
3
4
5
Perfectly
96
Q

HbS mutation

A

Non-polar valine amino acid replaces a negatively charged glutamate in the beta chain.

How well did you know this?
1
Not at all
2
3
4
5
Perfectly
97
Q

How do Na, K, Cl and Ca equilibrate when their respective channels are opened at physiologic conditions (membrane potential -70mV)

A

SODIUM: extracellular gradient flows into cell with starting equilibration potential of +60mV.

POTASSIUM: intracellular gradient flows out of cell with starting equilibration potential of -90mV.

CHLORIDE: extracellular gradient drives Cl- into the cell with starting equilibration potential of -75mV.

CALCIUM: extracellular gradient drives Ca2+ into the cell with a starting equilibrium potential of +125mV.

How well did you know this?
1
Not at all
2
3
4
5
Perfectly
98
Q

Colchicine mechanism of action

A

Inhibition of PMN microtubules prevents their translocation to the site of infection

How well did you know this?
1
Not at all
2
3
4
5
Perfectly
99
Q

Pathophysiology of Duchenne muscular dystrophy

A

X-linked recessive mutation in dystrophin gene resulting in myonecrosis and replacement with fibrofatty connective tissue.

How well did you know this?
1
Not at all
2
3
4
5
Perfectly
100
Q

How does cortisol exert its action on target cells?

A

It diffuses through the plasma membrane, binds to the cortisol receptor that is attached to a heat shock protein, the heat shock protein dissociates, two cortisol receptors bound by cortisol dimerize, the dimer translocates to the nucleus and modifies gene transcription.

How well did you know this?
1
Not at all
2
3
4
5
Perfectly
101
Q

How does glucagon exert its action on target cells?

A

G-protein coupled receptor

How well did you know this?
1
Not at all
2
3
4
5
Perfectly
102
Q

How does growth hormone exert its action on target cells?

A

Membrane bound receptors that result in activation of the JAK-STAT pathway

How well did you know this?
1
Not at all
2
3
4
5
Perfectly
103
Q

Ribavirin mechanism of action

A

1) It is phosphorylated and incorporated into RNA-dependent RNA replication and pairs with U and C
2) Ribavirin triphosphate directly inhibits HCV RNA polymerase
3) Ribavirin monophosphate inhibits intracellular inosine monophosphate dehydrogenase, depleting intracellular cGMP
4) Prevents viral RNA translation by inhibiting RNA guanylyl transferase and methyltransferase from forming an effective 5’ cap on mRNA
5) Enhances Th1 cell-mediated immunity and inhibits Th2 cytokine production

How well did you know this?
1
Not at all
2
3
4
5
Perfectly
104
Q

Raltegravir mechanism of action

A

HIV integrase inhibition, preventing integration of viral genes into host DNA

How well did you know this?
1
Not at all
2
3
4
5
Perfectly
105
Q

Indinavir mechanism of action

A

HIV protease inhibition

How well did you know this?
1
Not at all
2
3
4
5
Perfectly
106
Q

Amantadine mechanism of action

A

Prevents uncoating of influenza A virion after endocytosis

How well did you know this?
1
Not at all
2
3
4
5
Perfectly
107
Q

Enfuvirtide mechanism of action

A

HIV fusion inhibitor

How well did you know this?
1
Not at all
2
3
4
5
Perfectly
108
Q

Neurons that mediate the stretch reflex

A

Intrafusal fiber -> 1a sensory axon -> synapses on spinal cord alpha motor neuron -> contraction

How well did you know this?
1
Not at all
2
3
4
5
Perfectly
109
Q

Neurons that prevent muscular damage when too much force is exerted in a muscle

A

Golgi tendon oran -> 1b sensory axon -> synapses on inhibitory spinal interneuron -> inhibits spinal cord alpha-motor neuron -> sudden relaxation

How well did you know this?
1
Not at all
2
3
4
5
Perfectly
110
Q

A-delta sensory fibers communicate what stimuli?

A

Sharp pain and temperature. Myelinated fibers transmit touch, proprioception and vibration from Pacinian and Ruffini corpuscles.

How well did you know this?
1
Not at all
2
3
4
5
Perfectly
111
Q

Pacinian corpuscles communicate what stimuli?

A

Rapidly adapting mechanoreceptors (touch, proprioception and vibration), located in subcutaneous tissue, mesentery, peritoneum and joint capsules.

How well did you know this?
1
Not at all
2
3
4
5
Perfectly
112
Q

Ruffini corpuscles communicate what stimuli?

A

Slowly adapting mechanoreceptors (touch, proprioception and vibration), located in subcutaneous tissue and joints.

How well did you know this?
1
Not at all
2
3
4
5
Perfectly
113
Q

Giant cell arteritis histology

A

Granulomatous inflammation of media and fragmented internal elastic lamina in arteries of the head and neck.

How well did you know this?
1
Not at all
2
3
4
5
Perfectly
114
Q

How can you differentiate acute precursor T-cell ALL from acute precursor B-cell ALL?

A

Precursor B-ALL: TdT+, CD10 and almost always CD19

Precursor T-ALL: TdT+, +/- CD1a, CD2, CD3, CD4, CD5, CD7 or CD8

How well did you know this?
1
Not at all
2
3
4
5
Perfectly
115
Q

Diagnosis in a patient with enlarged, non-tender cervical lymph node that showed cells with abundant cytoplasm, bilobed nuclei and inclusion-like eosinophilic nucleoli

A

Reed-Sternberg cells -> Hodgkin lymphoma

How well did you know this?
1
Not at all
2
3
4
5
Perfectly
116
Q

What is responsible for the atypical lymphocytes seen in EBV infection?

A

EBV binds CD21 on B-cells and infects them. The atypical cells are T-cells that underwent clonal expansion and kill infected B-cells.

How well did you know this?
1
Not at all
2
3
4
5
Perfectly
117
Q

What makes up the class I and II MHC molecules

A

MHC I: Beta-2 micro globulin and a heavy chain

MHC II: alpha and beta polypeptide chains

How well did you know this?
1
Not at all
2
3
4
5
Perfectly
118
Q

Lab findings associated with post-strep glomerulonephritis

A

Elevated ASO titers, anti-DNase antibodies, presence of cryoglobulins and decreased C3 and total complement levels (C4 is typically normal).

How well did you know this?
1
Not at all
2
3
4
5
Perfectly
119
Q

Most common visual change in a PCA stroke

A

Contralateral hemianopia with macular sparing due to collateral circulation from MCA

How well did you know this?
1
Not at all
2
3
4
5
Perfectly
120
Q

Consequence of a stroke in the artery of Percheron

A

Bilateral thalamic or dorsal midbrain strokes

How well did you know this?
1
Not at all
2
3
4
5
Perfectly
121
Q

Pathophysiology behind maple syrup urine disease

A

Defective breakdown of branched chain amino acids (leucine, isoleucine, valine) due to non-functioning branched chain alpha-keto acid dehydrogenase.

How well did you know this?
1
Not at all
2
3
4
5
Perfectly
122
Q

Differences between enhancers/repressors and promotors of DNA transcription?

A

Promotors: TATA box is 25 nucleotides upstream from the start codon and facilitates RNA polymerase II and transcription factor binding. The CAAT box has a similar function and is 70 base pairs upstream.

Enhancers/repressors: can be located anywhere and increase or decrease the rate of transcription initiation

How well did you know this?
1
Not at all
2
3
4
5
Perfectly
123
Q

Power of a study

A

Probability of seeing a difference in your study when there truly is one present:

1 - probability of making a type II error (concluding there is no difference when there truly is one)

How well did you know this?
1
Not at all
2
3
4
5
Perfectly
124
Q

Type I error

A

Probability that you would see a difference when there really is not a difference…correlates with the p-value

How well did you know this?
1
Not at all
2
3
4
5
Perfectly
125
Q

Phases of acute tubular necrosis

A

Initiation: ~36 hours, mild drop in urine production as renal tubules begin to be damaged.

Maintenance: 1- 2 weeks, oliguria, reduced GFR, biopsy shows epithelial necrosis, tubular BM, casts with debris

Recovery: GFR recovers as the tubules re-epithelialize, tubules are slower to start recovering solutes, resulting in polyuric phase

How well did you know this?
1
Not at all
2
3
4
5
Perfectly
126
Q

Blood supply to the heart

A

LAD: anterior 2/3 of inter ventricular septum, anterior LV wall and anterior papillary muscle

LCX: lateral and posterior superior walls of LV

RCA: posterior (via acute marginal branches) and inferior heart (via posterior descending artery)

How well did you know this?
1
Not at all
2
3
4
5
Perfectly
127
Q

Bioavailability

A

Fraction of drug that reaches systemic circulation unchanged

Area under oral curve x IV dose)/(area under IV curve x oral dose

How well did you know this?
1
Not at all
2
3
4
5
Perfectly
128
Q

Disaccharide breakdown

A

Sucrose: fructose + glucose
Lactose: galactose + glucose by brush border enzyme lactase (beta-galactosidase)
Maltose: glucose + glucose

How well did you know this?
1
Not at all
2
3
4
5
Perfectly
129
Q

Pathophysiology of hereditary fructose intolerance

A

Aldolase B deficiency results in decreased conversion of fructose-1-P to DHAP and glyceraldehyde. Excess fructose-1-P results in intracellular phosphate depletion, inactivation of hepatic phosphorylase and reduced gluconeogenesis.

Impaired gluconeogenesis results in severe hypoglycemia after eating sucrose or fructose (fruit or formula). Infants are typically find while breastfeeding because breast milk contains mostly maltose and lactose.

How well did you know this?
1
Not at all
2
3
4
5
Perfectly
130
Q

Babies that present in the first few days of life with vomiting, jaundice, poor feeding and hepatomegaly may have what problem?

A

Galactosemia with reduced galactose metabolism.

How well did you know this?
1
Not at all
2
3
4
5
Perfectly
131
Q

How does leucovorin have a different effect when paired with methotrexate vs. 5-FU?

A

With methotrexate, it results in rescue of cellular replication because it is a tetrahydrofolate derivative and replenishes depleted tetrahydrofolate from MTX inhibition of tetrahydrofolate reductase.

With 5-FU, thymidylate synthase requires tetrahydrofolate in order to bind 5-FU and leucovorin potentiates the effects of 5-FU, reducing cellular replication.

How well did you know this?
1
Not at all
2
3
4
5
Perfectly
132
Q

Strep pyogenes major virulence factor

A

Protein M: inhibits phagocytosis and complement activation. It is also cytotoxic to PMNs and promotes bacterial aggregation.

How well did you know this?
1
Not at all
2
3
4
5
Perfectly
133
Q

Hormone levels seen in patients with Klinefelter’s

A

These patients typically have absent Sertoli cells, resulting in no production of inhibin and increased FSH levels.

They also have reduced Leydig cell function, resulting in reduced testosterone production and increased LH and GnRH levels.

How well did you know this?
1
Not at all
2
3
4
5
Perfectly
134
Q

Why do patients who use steroids develop small testes?

A

Exogenous testosterone levels inhibit LH release from the pituitary. This results in decreased testosterone production by Leydig cells and atrophy of local seminiferous tubules.

How well did you know this?
1
Not at all
2
3
4
5
Perfectly
135
Q

Treatment of restless leg syndrome

A

Limit exposure to aggravating factors (SSRIs, Li, glucocorticoids), supplement iron if anemic and dopamine agonists if needed (ropinirole or pramipexole)

How well did you know this?
1
Not at all
2
3
4
5
Perfectly
136
Q

Causes of splenomegaly

A

Increased white pulp mass from increased antibody production, B-cell affinity maturation or removal of opsonized material.

Increased red pulp mass from increased removal of RBCs or sinusoidal dilation from portal hypertension

How well did you know this?
1
Not at all
2
3
4
5
Perfectly
137
Q

Why would someone with pyruvate kinase deficiency have an enlarged spleen?

A

RBCs do not have mitochondria and any deficiency in glycolysis results in insufficient ATP generation for the RBC and inability to maintain the RBC membrane. This results in splenic sequestration of damaged RBCs.

How well did you know this?
1
Not at all
2
3
4
5
Perfectly
138
Q

Why give diuretics and beta blockers when prescribing hydralazine or minoxidil?

A

They prevent the reflex sympathetic activation resulting in tachycardia, increased contractility and renin activation leading to Na+ retention and edema.

How well did you know this?
1
Not at all
2
3
4
5
Perfectly
139
Q

How do glucocorticoids increase blood sugar levels?

A

They increase formation of gluconeogenic enzymes PEP carboxylase and glc-6 phosphatase. They increase glycogenesis by increasing formation of glycogen synthase. Finally, they inhibit insulin’s peripheral effect on muscle and fat, resulting in catabolism of these tissues to generate substrates for glycogenesis and gluconeogenesis.

How well did you know this?
1
Not at all
2
3
4
5
Perfectly
140
Q

A patient presents with acute, painless monocular vision loss and a cherry red macula. What is causing his condition?

A

Central retinal artery occlusion

How well did you know this?
1
Not at all
2
3
4
5
Perfectly
141
Q

Calculate PAO2. Normal A-a gradient?

A

150 - (PaCO2 / 0.8). PAO2 - PaO2 is normally 10-15

How well did you know this?
1
Not at all
2
3
4
5
Perfectly
142
Q

What is the difference between true and false diverticula?

A

True: all layers of wall to include mucosa, muscular and serosa.

False: mucosa, submucosa and serosa, but no muscular wall herniation

How well did you know this?
1
Not at all
2
3
4
5
Perfectly
143
Q

Major manifestations of acute rheumatic fever

A
Joint pain
Heart murmur
Nodules under skin
Erythema marginaum
Sydenham chorea
How well did you know this?
1
Not at all
2
3
4
5
Perfectly
144
Q

Phases of lyme disease

A

Early localized: flu-like symptoms + erythema migrans
Early disseminated: facial palsy, nodal block
Late disseminated: arthritis, subacute encephalopathy

How well did you know this?
1
Not at all
2
3
4
5
Perfectly
145
Q

2nd line treatment for lime disease

A

Beta-lactams like ceftriaxone

How well did you know this?
1
Not at all
2
3
4
5
Perfectly
146
Q

Colchicine mechanism of action

A

Binds intracellular tubulin and inhibits microtubule polymerization. This prevents PMN phagocytosis of crystals and chemotaxis to the site of inflammation.

How well did you know this?
1
Not at all
2
3
4
5
Perfectly
147
Q

Test to diagnose someone with Prinzmetals angina

A

Ergonovine (alpha-1 and 5-HT agonist) stimulation test will cause coronary vasospasm and chest pain at very low doses in these patients.

How well did you know this?
1
Not at all
2
3
4
5
Perfectly
148
Q

Why treat heart failure patients with neurolypsin inhibitors?

A

Neurolypsin is a MMP that degrades ANP and BNP. Inhibiting neurolypsin increases the activity of ANP and BNP resulting in vasodilation, renin inhibition and natriuresis.

How well did you know this?
1
Not at all
2
3
4
5
Perfectly
149
Q

How does brown fat generate heat?

A

It is rich in mitochondria that contain the uncoupling protein thermogenin.

How well did you know this?
1
Not at all
2
3
4
5
Perfectly
150
Q

Location of foregut, midgut and hindgut structures and blood supply?

A

Foregut: mouth to 3rd part of duodenum (celiac)
Midgut: 3rd part of duodenum to proximal 2/3 of transverse colon (SMA)
Hindgut: distal 2/3 of transverse colon to anus (IMA)

How well did you know this?
1
Not at all
2
3
4
5
Perfectly
151
Q

Lab that could diagnose someone with Wernicke encephalopathy (not imaging)?

A

B1 deficiency results in poor glucose metabolism because B1 is required for pyruvate dehydrogenase, alpha-ketoglutarate dehydrogenase and transketolase to function. Measuring erythrocyte transketolase can be diagnostic for Wernicke encephalopathy.

How well did you know this?
1
Not at all
2
3
4
5
Perfectly
152
Q

Causes of galactosemia

A

Type I) GALT deficiency
Type II) galactokinase deficiency
Type III) UDP-galactose-4-epipermase deficiency

How well did you know this?
1
Not at all
2
3
4
5
Perfectly
153
Q

Most common cause of hypoketotic hypoglycemia

A

Acyl CoA dehydrogenase deficiency the prevents progression of beta-oxidation of the respective VL chain, long chain, medium chain or short chain fatty acids metabolized by the enzyme.

How well did you know this?
1
Not at all
2
3
4
5
Perfectly
154
Q

Pathophysiology of Huntington’s disease

A

CAG repeats result in increased cell death in the striatum (caudate and putamen) due to increased glutamate binding to NMDA receptors.

How well did you know this?
1
Not at all
2
3
4
5
Perfectly
155
Q

Pathogens most likely responsible for secondary bacterial pneumonia

A

S. pneumo > S. aureus > H. influenza

How well did you know this?
1
Not at all
2
3
4
5
Perfectly
156
Q

Types of Arnold-Chiari malformations

A

I) tonsils extend into foramen magnum and cause headaches and ataxia as an adult

II) cerebellar vermis and medulla extend into foramen magnum causing aqueduct stenosis, hydrocephalus and meningomyelocele as a neonate

How well did you know this?
1
Not at all
2
3
4
5
Perfectly
157
Q

How can you prevent TPN-induced gallstone formation?

A

Administer cholecystokinin to increased biliary motility

How well did you know this?
1
Not at all
2
3
4
5
Perfectly
158
Q

Treatment of cryptococcal meningitis

A

Amphotericin B and flucytosine during initial period. Long-term maintenance therapy with fluconazole.

How well did you know this?
1
Not at all
2
3
4
5
Perfectly
159
Q

HPV strains responsible for disease

A

1-4: verrucas vulgaris (skin warts)
6, 11: condyloma acuminatum (genital warts)
16, 18, 31: over expression of viral oncogenes E6 (degrades p53) and E7 (inhibits RB1 tumor suppression) leading to cervical, vaginal, vulvar and anal cancer

How well did you know this?
1
Not at all
2
3
4
5
Perfectly
160
Q

Diphenoxylate mechanism of action

A

Opioid analogue similar to meperidine and used to treat diarrhea. It is complexed with atropine to prevent abuse.

How well did you know this?
1
Not at all
2
3
4
5
Perfectly
161
Q

Drug helpful for secretory diarrhea

A

Bismuth subsalicylate, octreotide and probiotics

How well did you know this?
1
Not at all
2
3
4
5
Perfectly
162
Q

Mechanism and side effects of pioglitazone and rosiglitazone

A

It is a thiazolidinedione, which means it binds PPAR-gamma, which complexes with RXR and results in increased insulin sensitivity in muscle and liver, decreased leptin production, increased fatty acid uptake, increased adiponectin production and decreased TNF-alpha production.

Side effects are weight gain and fluid retention.

How well did you know this?
1
Not at all
2
3
4
5
Perfectly
163
Q

PCN vs. vancomycin mechanism of action

A

Covalently binds and inhibits bacterial transpeptidase. This prevents the enzyme from cross linking an amino acid of the 3rd peptidoglycan molecule to the terminal D-ala-D-ala of another peptidoglycan molecule.

Vancomycin directly binds the D-ala-D-ala and inhibits incorporation of new subunits into the bacterial wall

How well did you know this?
1
Not at all
2
3
4
5
Perfectly
164
Q

What type of virus is parvovirus? EBV? adenovirus? HCV? HEV? HIV? orthomyxoviruses (influenza)? paramyxoviruses (parainfluenza, RSV, measles and mumps)? rhinovirus? reoviruses (rotavirus)?

A
Parvovirus: non-enveloped single-stranded DNA
EBV: enveloped ds-DNA
Adenovirus: non-enveloped ds-DNA
HCV: enveloped +ssRNA
HEV: non-enveloped +ssRNA
HIV: enveloped +ssRNA
Influenza: enveloped -ssRNA
Paramyxovirus: enveloped -ssRNA
Rhinovirus: non-enveloped +ssRNA
Rotavirus: dsRNA
How well did you know this?
1
Not at all
2
3
4
5
Perfectly
165
Q

Major functions of the hypothalamic nuclei

A

Ventromedial: satiety, destruction = ventral & medial waist growth

Lateral: hunger, destruction = anorexia

Anterior: A/C, destruction = hyperthermia

Posterior: heater, destruction = hypothermia

Arcuate: secretes DA, GhRH and GnRH

Paraventricular: secretes ADH, CRH, oxytocin and TRH

Supraoptic: secretes ADH & oxytocin

Suprachiasmatic: regulates circadian rhythm and regulation of the pineal gland (responsible for elevated melatonin at night and elevated cortisol in morning)

How well did you know this?
1
Not at all
2
3
4
5
Perfectly
166
Q

Malignancies associated with paraneoplastic dermatomyositis

A

Ovarian, lung, colon and non-Hodgkin lymphoma

How well did you know this?
1
Not at all
2
3
4
5
Perfectly
167
Q

Brain tumor biopsy shows….
A) psammoma bodies
B) nests of palisading squamous epithelium and areas of wet keratin
C) perivascular rosettes
D) small blue cells that surround neuropil, forming Homer-Wright rosettes
E) pseudopalisading atypical astrocytes bordering areas of necrosis
F) fried-egg appearance
G) eosinophilic granular bodies with elongated hairlike processes (Rosenthal bodies)
H) spindle cells with palisading nuclei around Verocay bodies composed of eosinophilic cores (Antoni A pattern)

A
A) meningioma
B) craniopharyngioma
C) ependymoma
D) medulloblastoma
E) glioblastoma
F) oligodendrocytoma
G) pilocytic astrocytoma
H) Schwannoma
How well did you know this?
1
Not at all
2
3
4
5
Perfectly
168
Q

Bacteriology of gardnerella vaginalis

A

Anaerobic gram variable rod

How well did you know this?
1
Not at all
2
3
4
5
Perfectly
169
Q

Gastric parietal cells secrete HCl in response to…

A

1) enterochromaffin-like cells release histamine that binds H2 receptors on parietal cells
2) vagal stimulation results in ACh binding M3 receptors, causing direct HCl release. It also causes release of gastrin releasing peptide, which stimulates gastrin release from G cells
3) protein-rich material causes gastrin release from G-cells. Gastrin binds to CCKa receptor on enterochromaffin-like cells and increases histamine release.

How well did you know this?
1
Not at all
2
3
4
5
Perfectly
170
Q

Lung met with clear cells and elevated Hct vs. clear cells with no change in Hct and early satiety?

A

Elevated Hct = renal clear cell carcinoma

Early satiety = stomach signet cell carcinoma

How well did you know this?
1
Not at all
2
3
4
5
Perfectly
171
Q

Oseltamivir vs. amantadine

A

Oseltamivir: neuraminidase inhibitor, prevents virion release from the infected cell

Amantadine: prevents viral disassembly after endocytosis

How well did you know this?
1
Not at all
2
3
4
5
Perfectly
172
Q

Method of hepatic metabolism for INH, hydralazine, dapsone and procainamide. People with decreased metabolism of this type are at risk for what condition?

A

Hepatic microsomal acetylation by N-acetyl-transferase that make them more soluble and able to be excreted in the urine. Patients who are slow acetylators are at increased risk for drug-induced Lupus.

How well did you know this?
1
Not at all
2
3
4
5
Perfectly
173
Q

Method of hepatic metabolism for 6-MP and azathioprine

A

Methylation

How well did you know this?
1
Not at all
2
3
4
5
Perfectly
174
Q

Molecular activation that occurs when EPI binds to its alpha-1 receptor?

A

GDP -> GTP on the alpha-subunit of the G-protein -> alpha-subunit dissociates and exposes its catalytic subunit for PLC -> IP3 + DAG release from plasma membrane -> Ca release from sarcoplasmic reticulum (IP3) and protein kinase C activation (DAG + Ca) -> protein phosphorylation

How well did you know this?
1
Not at all
2
3
4
5
Perfectly
175
Q

Post-transcription processing of RNA in eukaryotes prior to translation

A

5’ cap: guanine triphosphate is added to 5’ end by guanylyltransferase -> guanine triphosphate is methylated by guanine-7-methyltransferase -> mRNA is protected from exonucleases and can exit nucleus

Poly-A tail: polyadenlyate polymerase complex recognizes AAUAAA sequence at the end of mRNA, cleaves the mRNA after this sequence and then adds 20-250 adenosine residues in the nucleus. This tail is gradually shortened in the cytosol, leading to mRNA degradation.

Intron splicing occurs before leaving the nucleus.

In the cytosol, some mRNAs associate with ribosomes immediately and others are bound by P proteins for later use

How well did you know this?
1
Not at all
2
3
4
5
Perfectly
176
Q

Viruses that acquire their envelopes by budding through the nuclear membrane

A

Herpesviruses

How well did you know this?
1
Not at all
2
3
4
5
Perfectly
177
Q

Why vitamin K deficiency increases propensity to bleed

A

It is needed for the enzyme gamma-glutamyl carboxylase, which creates a Ca2+ binding site on factors II, VII, IX and X that is necessary to bind Ca2+ to bind to negatively charged phospholipids and platelets on damaged endothelial cells.

How well did you know this?
1
Not at all
2
3
4
5
Perfectly
178
Q

Value that indicates fetal lung maturity

A

Lethicin (phosphatidylcholine) : sphingomyelin ratio > 2

How well did you know this?
1
Not at all
2
3
4
5
Perfectly
179
Q

How are sound waves transmitted into nervous impulses that can be interpreted by the brain?

A

TM vibration -> ossicles transmit vibration to oval window -> basilar membrane vibrates -> hair cell cilia bend against tectorial membrane in the organ of Corti -> oscillating hyper polarization and depolarization of the auditory nerve

How well did you know this?
1
Not at all
2
3
4
5
Perfectly
180
Q

Acoustic reflex to loud sound

A

The stapedius and tensor tympani contract to decrease vibration transmitted from the ossicles.

How well did you know this?
1
Not at all
2
3
4
5
Perfectly
181
Q

Class I antiarrhythmics with most prominent use-dependent effects?

A

IC (slow dissociation from Na-channel leads to cumulative effect) > IA (intermediate) > IB (rapid dissociation from Na-channel leads to negligible cumulative effect)

How well did you know this?
1
Not at all
2
3
4
5
Perfectly
182
Q

Rasburicase mechanism of action

A

Recombinant urate oxidase, which is not typically present in humans. It converts uric acid to allantoin, which is more soluble and useful in treatment of tumor lysis syndrome.

How well did you know this?
1
Not at all
2
3
4
5
Perfectly
183
Q

Denosumab mechanism of action

A

Monoclonal antibody that prevents RANK-L from activating osteoclasts in treatment of osteoporosis.

How well did you know this?
1
Not at all
2
3
4
5
Perfectly
184
Q

Contraindication to administration of cholestyramine for high cholesterol

A

It increases liver production of VLDL and triglycerides, so don’t give to a patient with hypertriglyceridemia. It also increases cholesterol content of bile, so don’t give to a patient with gallstones.

How well did you know this?
1
Not at all
2
3
4
5
Perfectly
185
Q

Pathophysiology of Eisenmenger syndrome

A

Elevated right atrial pressure from L -> R shunting results in laminar medial hypertrophy of the muscular pulmonary arteries -> elevated pulmonary vascular resistance > systemic vascular resistance -> R->L shunting of blood. At this time, the shunt is permanent and can no longer be closed.

How well did you know this?
1
Not at all
2
3
4
5
Perfectly
186
Q

Extra-skeletal manifestations of ankylosing spondylitis

A

Hypoventilation from limited chest wall expansion
Ascending aortitis -> AI
Anterior uveitis

How well did you know this?
1
Not at all
2
3
4
5
Perfectly
187
Q

3 mechanisms of bacterial gene transfer

A

1) Transformation: direct uptake of naked DNA, typically by S. pneumo, H. influenzae and Neisseria
2) Conjugation: donor bacteria has DNA F-factor that codes for sex pilus to transfer DNA to recipient bacteria
3) Transduction: random bacterial genes are accidentally packaged into viral capsid during lytic stage, then the virus (bacteriophage) infects another bacteria and integrates the donor DNA into the recipients DNA

How well did you know this?
1
Not at all
2
3
4
5
Perfectly
188
Q

Pathophysiology of TTP. Treatment?

A

Low ADAMTS13 results in large vWF multimers that cause formation of diffuse, platelet-rich thrombi to form in microvasculature. This can be inherited or acquired and results in microangiopathic hemolytic anemia with schistocytes and thrombocytopenia with increased bleeding time and normal PT/aPTT. Treat with plasma exchange.

How well did you know this?
1
Not at all
2
3
4
5
Perfectly
189
Q

Regions most prone to developing atherosclerotic lesions

A

Coronary arteries and abdominal aorta

How well did you know this?
1
Not at all
2
3
4
5
Perfectly
190
Q

Virulence factors associated with traveler’s diarrhea

A

ETEC has heat labile (LT) and heat stable (ST) enterotoxins. On colonization of the small intestine by pili adherence, LT activates Gs protein -> increased cAMP. ST binds guanylate cyclase on apical membranes and increases cGMP. cAMP and cGMP increase gut mucosa secretion of water and electrolytes into the lumen resulting in a secretory diarrhea.

How well did you know this?
1
Not at all
2
3
4
5
Perfectly
191
Q

An infant has persistent diarrhea. Small intestine biopsy shows gram negative rods in a stacked brick appearance. What is the bug?

A

EAEC

How well did you know this?
1
Not at all
2
3
4
5
Perfectly
192
Q

Areas of the brain most prone to infarction in sustained periods of hypotension

A

Purkinje cells of the cerebellum and pyramidal cells of the hippocampus. Watershed areas are also susceptible.

How well did you know this?
1
Not at all
2
3
4
5
Perfectly
193
Q

A new child is born with cleft lip, cleft palate, polydactyly, rocker-bottom feet and holoprosencephaly. What is the chromosomal abnormality?

A

Patau syndrome is due to non-disjunction of chromosome 13 during the maternal phase of meiosis I.

How well did you know this?
1
Not at all
2
3
4
5
Perfectly
194
Q

A new child is born with a small jaw, prominent occiput, low set ears, rocker-bottom feet and a small mouth. Exam reveals clenched hands with the index finger overriding the middle finger and the pinky overriding the ring finger. What is the chromosomal abnormality?

A

Edwards syndrome.

How well did you know this?
1
Not at all
2
3
4
5
Perfectly
195
Q

Innervation, lymphatics and blood supply above and below the dentate line

A

Above: inferior hypogastric plexus (autonomic, only sensitive to stretch and pressure), drains to inferior mesenteric and internal iliac lymph nodes, blood supply from superior rectal and middle rectal arteries.

Below: inferior rectal nerve, a branch of the pudendal and is exquisitely sensitive to pain, temperature and light touch. Lymph drainage is to superficial inguinal nodes.

How well did you know this?
1
Not at all
2
3
4
5
Perfectly
196
Q

Mechanism of alpha and beta interferon in treatment of hepatitis C and other viral infections?

A

They bind to type I interferon receptors on infected and neighboring cells. This results in transcription of antiviral enzymes RNase L (endonuclease that degrades RNA) and protein kinase R (inactivates eIF-2, preventing translation of viral RNA). As a result normal metabolism occurs in uninfected cells and translation is inhibited in infected and proximal cells.

They also increase MHC I expression on all cells, increase NK and cytotoxic T-cell activity

How well did you know this?
1
Not at all
2
3
4
5
Perfectly
197
Q

Treatment of severe hypoglycemia out of the hospital setting

A

IM glucagon will return the patient to consciousness within 10-15 minutes. In the hospital you give IV 50% dextrose.

How well did you know this?
1
Not at all
2
3
4
5
Perfectly
198
Q

Viral phenotypic mixing

A

A cell is coinfected with virus A and virus B resulting in expression of glycoproteins on the capsid or envelope from both viruses allowing them to infect cells they would not otherwise infect. This is phenotypic because there is no exchange of genetic material and the next generation will have the original envelope or capsid again if it is the only virus infecting the cell. Contrast this to reassortment and recombination where the next generation of virions remains changed.

How well did you know this?
1
Not at all
2
3
4
5
Perfectly
199
Q

Self-limited pharyngoconjuntival fever in military recruits

A

Adenovirus

How well did you know this?
1
Not at all
2
3
4
5
Perfectly
200
Q

Febrile aseptic meningoencephalitis in patients exposed to hamsters or mice?

A

LCMV, an arenavirus

How well did you know this?
1
Not at all
2
3
4
5
Perfectly
201
Q

Prerequisite for hospice care

A

Prognosis

How well did you know this?
1
Not at all
2
3
4
5
Perfectly
202
Q

Test to distinguish tuberculoid from lepromatous leprosy

A

Lepromin skin test will be + in patients with tuberculoid leprosy because the generate a robust Th1 cytokine profile that activates macrophages and Th1 cells.

How well did you know this?
1
Not at all
2
3
4
5
Perfectly
203
Q

Vincristine and vinblastine mechanism of action

A

Bind B-tubulin and inhibit microtubule polymerization

How well did you know this?
1
Not at all
2
3
4
5
Perfectly
204
Q

Patient on chemo gets flagellated skin discoloration and pulmonary fibrosis

A

Bleomycin

How well did you know this?
1
Not at all
2
3
4
5
Perfectly
205
Q

What do 99% and 95% confidence intervals mean?

A

They represent standard error which is the corresponding z-score divided by the square root of “n”…i.e. people in the study.

99% = mean +/- (2.58 x SD) / square root of n
95% = mean +/- (1.96 x SD) / square root of n
How well did you know this?
1
Not at all
2
3
4
5
Perfectly
206
Q

Why are patients with SIADH euvolemic?

A

The mild initial increase in volume promotes natriuresis and normalization of volume status with continued dilutional hyponatremia.

How well did you know this?
1
Not at all
2
3
4
5
Perfectly
207
Q

Cytokines with anti-inflammatory properties

A

IL-10: it reduces Th1 cytokine production (IL-2 and IFN-gamma), inhibits activated dendritic cells and macrophages and reduces MHC II expression on antigen presenting cells.

How well did you know this?
1
Not at all
2
3
4
5
Perfectly
208
Q

Bortezomid mechanism of action

A

Boronic acid-containing dipeptide proteosome inhibitor that is good for treating high protein output conditions like multiple myeloma because it results in accumulation of pro-apoptotic misfolded and damaged proteins within the neoplastic plasma cells.

How well did you know this?
1
Not at all
2
3
4
5
Perfectly
209
Q

Most common cause of unexpected mortality after cardiac catheterization?

A

Retroperitoneal hemorrhage that occurs secondary to cannulation of the femoral artery above the inguinal ligament

How well did you know this?
1
Not at all
2
3
4
5
Perfectly
210
Q

Mnemonic for class I antiarrhythmics

A

IA) “Double quarter pounder: disopyramide, quinidine and procainamide

IB) “Lettuce, tomato, mayo”: lidocaine, tocainide, mexilitine

IC) “More fries please”: moricizine, flecainide, propafenone

How well did you know this?
1
Not at all
2
3
4
5
Perfectly
211
Q

Different effects of class I anti-arrhythmics on the cardiac action potential? Class II?

A

IA: slowed phase 0 and prolonged phase 3
IB: minimal change in phase 0 and shortened phase 3
IC: drastic phase 0 slowing and minimal change in phase 3

II: slowed phase 4 depolarization

III: drastic prolongation of phase 3

IV: minimal effect on action potential

How well did you know this?
1
Not at all
2
3
4
5
Perfectly
212
Q

HIV mechanism of cell entry

A

HIV envelope glycoprotein gp120 binds CCR5 and CXCR4 -> endocytosis by membrane fusion

How well did you know this?
1
Not at all
2
3
4
5
Perfectly
213
Q

Calculating half-life of a drug after administration of a single dose

A

t1/2 = (0.7 x Vd) / CL

How well did you know this?
1
Not at all
2
3
4
5
Perfectly
214
Q

Conditions associated with microangiopathic hemolytic anemia

A

HUS, TTP and DIC due to widespread thrombosis that destroys RBCs

How well did you know this?
1
Not at all
2
3
4
5
Perfectly
215
Q

Aplastic anemia vs. myelofibrosis

A

Aplastic anemia: no splenomegaly and fatty marrow infiltrated

Myelofibrosis: splenomegaly and collagenous deposition for megakaryocyte platelet activation in marrow

How well did you know this?
1
Not at all
2
3
4
5
Perfectly
216
Q

Why give Nimodipine to a patient who presents with subarachnoid hemorrhage.

A

In addition to communicating hydrocephalus and rebelled, these patients are at risk for vasospasm and subsequent neurological sequelae. CCBs help reduce incidence of vasospasm.

How well did you know this?
1
Not at all
2
3
4
5
Perfectly
217
Q

Ortner syndrome

A

LA enlargement significant enough to cause horseness and recurrent laryngeal nerve impingement

How well did you know this?
1
Not at all
2
3
4
5
Perfectly
218
Q

Common anaerobic gram-negative bacillus isolated from intra-abdominal infections

A

Bacteroides

How well did you know this?
1
Not at all
2
3
4
5
Perfectly
219
Q

t(14;18)

A

Follicular lymphoma due to Bcl-2 activation and subsequent inhibition of apoptosis.

How well did you know this?
1
Not at all
2
3
4
5
Perfectly
220
Q

How do macrophages enhance their immune response when encountering bacteria?

A

Secrete IL-12 -> CD4 cells differentiate into Th1 cells and secrete IFN-gamma -> enhanced macrophage killing ability. Macrophages also secrete TNF-alpha to recruit other monocytes to the site of infection.

How well did you know this?
1
Not at all
2
3
4
5
Perfectly
221
Q

In the lung where is:
A) arterial pressure > alveolar pressure > venous pressure
B) arterial pressure > venous pressure > alveolar pressure
C) alveolar pressure > arterial pressure > venous pressure

A

A) Zone II, pulmonary capillaries open in pulsatile fashion
B) Zone III, pulmonary capillaries are open
C) Zone I, pulmonary capillaries are collapsed

How well did you know this?
1
Not at all
2
3
4
5
Perfectly
222
Q

Pulmonary clearance of particles:
A) 10-15 micrometers in size
B)

A

A) Trapped in upper respiratory tract
B) Cleared by alveolar macrophages
C) Mucociliary clearance (primary method of clearance)

How well did you know this?
1
Not at all
2
3
4
5
Perfectly
223
Q

Why do pneumoconioses result in pulmonary fibrosis

A

Macrophage phagocytosis of small particles results in release of PDGF and IGF that attract fibroblasts and causes collagen deposition.

How well did you know this?
1
Not at all
2
3
4
5
Perfectly
224
Q

Episodic vertigo with unilateral hearing loss and ear pain

A

Meniere’s disease: increased endolymph pressure and volume

How well did you know this?
1
Not at all
2
3
4
5
Perfectly
225
Q

Pathogenesis of pancreatitis

A

Acinar injury -> trypsin activation -> protease (autodigestion), elastase (vascular erosion), lipase and phospholipase (fat necrosis) activation.

How well did you know this?
1
Not at all
2
3
4
5
Perfectly
226
Q

Why do lactose intolerant patient develop decreased pH in the gut?

A

Bacteria ferment lactose to short chain fatty acids (acetate, butyrate and propionate)

How well did you know this?
1
Not at all
2
3
4
5
Perfectly
227
Q

Patient has bone pain. Biopsy shows haphazardly organized segments of lamellar bone with prominent cement lines in a mosaic pattern.

A

Paget’s disease of the bone.

How well did you know this?
1
Not at all
2
3
4
5
Perfectly
228
Q

Tommy John surgery repairs what ligament

A

Ulnar collateral

How well did you know this?
1
Not at all
2
3
4
5
Perfectly
229
Q

Patient with new holosystolic murmur and cardiac biopsy showing interstitial fibrosis, scattered macrophages and multinucleated giant cells forming an interstitial myocardial granuloma (Aschoff body). What other special cell types might you see?

A

Aschoff bodies are pathognomonic for acute rheumatic fever. You may also see macrophages with plump cytoplasm and central slender chromatin ribbons called Antischkow cells. These Aschoff bodies are significant because they eventual fibrose and cause mitral stenosis.

How well did you know this?
1
Not at all
2
3
4
5
Perfectly
230
Q

Complications of diphtheria infection

A

Suffocation from pseudomembrane expansion, myocarditis and neurotoxicity.

How well did you know this?
1
Not at all
2
3
4
5
Perfectly
231
Q

Non-motile gram-positive rods that clump in V and Y configurations and intracytoplasmic granules that stain with analine dye.

A

C. diphtheriae

How well did you know this?
1
Not at all
2
3
4
5
Perfectly
232
Q

Why does diphtheria have such a high affinity for cardiac cells?

A

The B part of the AB toxin binds the heparin-binding EGF-receptor on neurons and myocytes.

How well did you know this?
1
Not at all
2
3
4
5
Perfectly
233
Q

Why does our body have a difficult time generating an immune response against HCV?

A

Due to lack of 3’ -> 5’ RNA-dependent RNA polymerase proofreading activity, there is significant antigenic variability in envelope glycoproteins with each replication of the virus.

How well did you know this?
1
Not at all
2
3
4
5
Perfectly
234
Q

Drugs that inhibit dihyrofolate reductase

A

MTX, TMP and pyrimethamine (good for toxo)

How well did you know this?
1
Not at all
2
3
4
5
Perfectly
235
Q

Why use cilastin with imipenem?

A

It inhibit renal metabolism of imipenem by competitively inhibiting dihydropeptidase on the tubular brush border.

How well did you know this?
1
Not at all
2
3
4
5
Perfectly
236
Q

Bacitracin mechanism of action

A

Inhibits cell wall synthesis by blocking mucopeptide transfer to the cell wall

How well did you know this?
1
Not at all
2
3
4
5
Perfectly
237
Q

How can the pancreas cause bleeding fundic varices?

A

Pancreatitis and inflammation can cause splenic vein thrombosis as the splenic vein lies posterior to the pancreas. The short gastric arteries normally drain to the splenic vein and blockage of the splenic vein results in increased venous pressure and fundic varices.

How well did you know this?
1
Not at all
2
3
4
5
Perfectly
238
Q

Vein that drains esophageal blood into the vena cava

A

Azygos

How well did you know this?
1
Not at all
2
3
4
5
Perfectly
239
Q

Cause of cleft lip vs. cleft palate

A

Cleft lip: failure of maxillary prominence to fuse with intermaxillary segment

Cleft palate: palatine shelves fail to fuse with one another or with the primary palate.

How well did you know this?
1
Not at all
2
3
4
5
Perfectly
240
Q

Motile gram-negative rod, non-fermenter that is oxidase positive and produces pyocyanin

A

Pseudomonas

How well did you know this?
1
Not at all
2
3
4
5
Perfectly
241
Q

Function of splenic red pulp

A

Destroys aged RBCs, stores blood cells and platelets for when excess are needed and macrophages clear circulating bacteria and present them to B and T cells in the white pulp. ***B cells in the spleen produce 50% of all the body’s opsinizing immunoglobulin.

How well did you know this?
1
Not at all
2
3
4
5
Perfectly
242
Q

Antifungals that block synthesis of glucan

A

Caspofungin and other echinocandins block synthesis of 1,3-beta-D-glucan and prevent formation of new fungi. It is most active against aspergillus and candida

How well did you know this?
1
Not at all
2
3
4
5
Perfectly
243
Q

Antifungal that binds ergosterol and puts holes in the fungal membrane. Major side effect?

A

AmphoB. Good for cryptococcal meningitis. Watch out for renal toxicity because it can cause renovascular constriction and direct tubular toxicity leading to electrolyte abnormalities and anemia due to decreased EPO release. Also patients can get an acute reaction immediately after infusion.

How well did you know this?
1
Not at all
2
3
4
5
Perfectly
244
Q

Augmenting agent to amphoB for cryptococcal meningitis that inhibits fungal DNA and RNA synthesis in fungi

A

Flucytosine

How well did you know this?
1
Not at all
2
3
4
5
Perfectly
245
Q

Antifungal that accumulates in keratin-containing tissue and inhibits microtubule function

A

Griseofulvin

How well did you know this?
1
Not at all
2
3
4
5
Perfectly
246
Q

Antifungal that inhibits the enzyme 14alpha-demethylases’s conversion of lanosterol to ergosterol necessary to make fungal plasma membranes. Side effects?

A

Itraconazole. These are potent inhibitors of CYP450, so care should be taken when prescribing them to patients on warfarin, tacrolimus, cyclosporine or oral hypoglycemics. Also if patients are on CYP450 inducers (rifampin, phenytoin, carbamazepine, phenobarbital), the azole may not be as effective.

How well did you know this?
1
Not at all
2
3
4
5
Perfectly
247
Q

Antifungal that accumulates in the skin and nails and inhibits squalene epoxidase, leading to decreased ergosterol synthesis

A

Terbinafine

How well did you know this?
1
Not at all
2
3
4
5
Perfectly
248
Q

Morphologic progression of pneumonia

A

Colonization: 1st 24 hours, lobe is red, boggy and alveolar exudate contains mostly bacteria
Red hepatization: days 2-3, lobe is red, firm and alveolar exudate contains mostly RBCs, PMNs and fibrin
Gray hepatization: days 4-6, lobe is gray, firm and alveolar exudate contains mostly PMNs, fibrin and disintegrating RBCs
Resolution: enzymatic digestion of exudate restores normal architecture.

How well did you know this?
1
Not at all
2
3
4
5
Perfectly
249
Q

Mechanism and key side effect of chloramphenicol

A

This peptidyl transferase inhibitor (which binds to the 50S subunit) can cause reversible and irreversible aplastic anemia. It can also cause gray baby syndrome.

How well did you know this?
1
Not at all
2
3
4
5
Perfectly
250
Q

How does shigella invade GI mucosal tissue?

A

Initial uptake into the M-cells allows it to lyse the M-cell and spread along the basal surface of the GI mucosa from cell to cell via actin polymerization (like listeria)

How well did you know this?
1
Not at all
2
3
4
5
Perfectly
251
Q

Enteric bugs that can infiltrate the mesenteric lymph nodes and cause pseudoappendicitis from LN inflammation

A

Yersinia and salmonella

How well did you know this?
1
Not at all
2
3
4
5
Perfectly
252
Q

Serotonin syndrome vs. neuroleptic malignant syndrome

A

Serotonin syndrome = muscular hyperactivity + hyperreflexia

NMS = diffuse muscular hyperrigidity + bradyreflexia

How well did you know this?
1
Not at all
2
3
4
5
Perfectly
253
Q

Sacubitril MOA

A

Inhibits neprilysin to prevent the degradation of ANP and enhancing its effects on natriuresis (dilates afferent arteriole, reduces Na+ reabsorption and limits aldosterone secretion from adrenals) and vasodilation.

How well did you know this?
1
Not at all
2
3
4
5
Perfectly
254
Q

Main blood supply of the femoral head

A

Medial circumflex femoral arteries

How well did you know this?
1
Not at all
2
3
4
5
Perfectly
255
Q

Why should you never treat a patient with strongyloides with corticosteroids

A

After infection through the skin by filariaform larvae and migration through the lungs and down the esophagus to the GI tract, they develop into adult worms. These adult worms produce rhabtidiform (non-infectious) larvae that can be excreted in the stool or molt into filariaform larvae, causing autoinfection. The ability to autoinfect the host results in super infection. Giving the patient corticosteroids will inhibit their Th2 response (same with patients with HTLV-1) and superinfection will result in shock due to poor parasitic control by basophils and eosinophils.

How well did you know this?
1
Not at all
2
3
4
5
Perfectly
256
Q

Treatment of strongyloides

A

Ivermectin

How well did you know this?
1
Not at all
2
3
4
5
Perfectly
257
Q

Markers that stain small cell lung cancer

A

Chromogranin, neuron-specific enolase and synaptophysin.

How well did you know this?
1
Not at all
2
3
4
5
Perfectly
258
Q

Why are patients with N. meningitides at risk for septic shock?

A

The bacteria has the LOS (similar to LPS) on its membrane, which binds to TLR-4 and induces release of TNF-alpha, IL-1, IL6 and IL-8.

How well did you know this?
1
Not at all
2
3
4
5
Perfectly
259
Q

Patient has asymmetric, multifocal neuropathy with skin nodules, migratory pulmonary infiltrates, sinus abnormalities and p-ANCA anti-MPO antibodies.

A

Eosinophilic granulomatosis with polyangiitis (Churg-Strauss). The neuropathy occurs due to vasculitis involving the epineural blood supply.

How well did you know this?
1
Not at all
2
3
4
5
Perfectly
260
Q

Aspiration pneumonia vs. pneumonitis

A

Pneumonia: due to aspiration of oropharyngeal bacteria, fever onset days after, may progress to form an abscess (most common cause of lung abscess), needs clindamycin or beta-lactam + beta-lactamase.

Pneumonitis: due to aspiration of gastric content, respiratory distress occurs within hours, infiltrates may be bilateral, will resolve on its own

How well did you know this?
1
Not at all
2
3
4
5
Perfectly
261
Q

Why is spironolactone good for patients with heart failure?

A

Blocks aldosterone binding leading to K+ sparing diuresis and blockage of cardiac remodeling.

How well did you know this?
1
Not at all
2
3
4
5
Perfectly
262
Q

Patients at high risk for drug-induced lupus

A

Procainamide (especially slow-acetylators), isoniazid and hydralazine.

How well did you know this?
1
Not at all
2
3
4
5
Perfectly
263
Q

Differentiating drug-induced lupus from SLE

A

Drug induced lupus commonly has anti-histone bodies, which are present in ~50% of patients with SLE.
Anti ds-DNA antibodies are common in SLE, but rare in drug-induced lupus.

How well did you know this?
1
Not at all
2
3
4
5
Perfectly
264
Q

Use of folinic acid with MTX vs. 5-FU

A

With MTX = reversal of effects due to bypassing of dihydrofolate reductase, which is inhibited by MTX
With 5-FU = potentiates effects due to increased binding of thymidylate synthase

How well did you know this?
1
Not at all
2
3
4
5
Perfectly
265
Q

Medication used to decrease toxicity of platinum-containing and alkylating chemotherapy drugs?

A

Amifostine is a free radical scavenger and reduces nephrotoxicity and xerostomia caused by these drugs.

How well did you know this?
1
Not at all
2
3
4
5
Perfectly
266
Q

Drug used to mitigate anthracycline-related cardiotoxicity

A

Dexrazoxane, an iron-chelating agnet

How well did you know this?
1
Not at all
2
3
4
5
Perfectly
267
Q

Bronchiolitis obliterans in a lung transplant patient

A

Signifies chronic rejection due to lymphocytic infiltration, inflammation and fibrosis of the bronchial wall

How well did you know this?
1
Not at all
2
3
4
5
Perfectly
268
Q

Areas of the brain at highest risk for global cerebral ischemia

A

1) Pyramidal cells of the hippocampus

2) Purkinje cells of the cerebellum

How well did you know this?
1
Not at all
2
3
4
5
Perfectly
269
Q

How does the body hurt itself when metabolizing CCl4?

A

The P450 microsomal system creates free radicals when oxidizing CCl4. These cause lipid peroxidation and damage to cellular membranes.

How well did you know this?
1
Not at all
2
3
4
5
Perfectly
270
Q

CD4 count when you worry about CMV retinitis and MAI

A
How well did you know this?
1
Not at all
2
3
4
5
Perfectly
271
Q

Most common trigger of erythema multiforme

A

HSV

272
Q

Why is cellular swelling the first sign of myocyte injury in MI?

A

Decreased perfusion -> decreased ATP for membrane Na/K pump and sarcoplasmic Ca pump -> increased intracellular Na + Ca -> osmotic swelling

273
Q

Why does the mutation in FAP cause cancer?

A

APC maintains low levels of beta-catenin, which is oncogenic. When mutated, levels rise as well as risk for cancer.

274
Q

Tumor most common in LiFraumeni

A

p53 mutations lead to tumors of the breast, bone, adrenals and sarcoma

275
Q

Nerve that innervates the sternohyoid, sternothyroid and omohyoid muscles loops around what blood vessel?

A

Ansa cervicalis (C1-C3) loops around the internal jugular vein

276
Q

A patient’s biopsy shows elongated cells with regular oval nuclei that for palisading patterns with nuclear free zones (Verocay bodies). What stain would be positive in this tumor?

A

Verocay bodies are seen in the Antoni A pattern of Schwannoma (Antoni B has minimal cellularity). Schwannomas and melanomas stain positive for S-100.

277
Q

Only cranial nerve that cannot develop a Schwannoma

A

CN II

278
Q

Patient with AIDS has watery diarrhea and intestinal biopsy shows basophilic clusters on the mucosal surface

A

Cryptosporidium

279
Q

How to get dopamine to a Parkinson patient’s brain

A

Give L-DOPA so it can cross the BBB and be converted to dopamine in the CNS

280
Q

Why do fetuses sometimes develop transient hydronephros in utero?

A

Around week 4-5 the mesonephric duct gives off the ureteric bud. The bud develops into the collecting ducts and ureters. The mesonephros develops into the renal parenchyma and the two structures become the metanephros. Occasionally, before the ureteric bud cannalizes, the metanephros begins producing urine and there is transient hydronephrosis.

281
Q

Why does delayed type hypersensitivity take days to come about.

A

Dendritic cells present antigen to CD4 cells, CD4 cells differentiate into Th1 cells, Th1 cells activate macrophages which migrate to the original site of contact and induce an inflammatory response.

282
Q

Metabolic disturbances with salicylate toxicity

A

1) Respiratory alkalosis from stimulation of central respiratory centers
2) Mixed respiratory alkalosis + metabolic acidosis due to uncoupling of oxidative phosphorylation, increased lipolysis and TCA cycle inhibition.

283
Q

How does ectopic ACTH cause skin hyperpigmentation

A

ACTH has significant similarity to MSH and can bind the MSH receptor on melanocytes to increase melanin production.

284
Q

Causes of heterophile antibody-negative mono-like syndromes

A

HIV, CMV, Toxo and HHV-6

285
Q

Most common infection that precedes Guillain-Barre

A

Campylobacter

286
Q

A patient with renal failure presents with altered mental status, seizures, lactic acidosis and bright red blood coming from his IV site after receiving a prolonged infusion of sodium nitroprusside. How should you treat him?

A

Infusion of sodium nitroprusside can lead to cyanide accumulation, which depletes the sulfur necessary for its metabolism by rhodanese. It then inhibits cytochrome c oxidase in the ETC. The following are treatments.

Sodium nitrite: promotes formation of methemoglobin, which can then sequester cyanide as cyanmethemoglobin

Sodium thiosulfate: allows the liver to donate sulfur to convert cyanide to thiocyanide via the enzyme “rhodanese” and be excreted in urine

Hydroxycobalamin: cobalt binds cyanide and is excreted in the urine as cyanocobalamin

287
Q

Acyclovir mechanism of action. Why doesn’t it work as well for EBV and CMV?

A

Viral thymidine kinase phosphorylates it to its monophosphate form, then cellular kinases phosphorylate to its triphosphate form, which impairs replication by viral DNA polymerase.

It doesn’t work as well for EBV and CMV because they have different activating viral kinases.

288
Q

Pathogen involved in a patient with diarrhea and:
A) Many fecal erythrocytes and a few leukocytes
B) Fecal leukocytes with monocyte predominance
C) Fecal leukocytes with PMN predominance
D) Mucus and sloughed epithelial cells

A

A) E. coli O157:H7 (shiga-like toxin without mucosal invasion)
B) S. typhi
C) Inflammatory diarrhea: salmonella, shigella, campylobacter, EIEC, yersinia, C. difficile and Entamoeba
D) Watery: cholera, ETEC, B. cereus, S. aureus, giardia, cryptosporidium, cyclospora, microsporidium and some viruses

289
Q

Ecthyma gangrenosum

A

P. aeruginosa infection of arteries and veins with toxic release of exotoxin A (EF-2 ADP ribosylator), elastase, phospholipase C and pyocyanin.

290
Q

Mode of transmission of neurocysticercosis

A

Human eats pig with tenia solium in meat -> human poops out eggs and doesn’t wash hands -> other human eats eggs -> cysts in brain

291
Q

Most common cause of death in patients who overdose on TCAs

A

TCAs inhibit fast Na+ conduction. This prolongs phase 0 in the cardiac action potential, prolonging the QRS and negative inotropic effects. This impairs excitation-contraction coupling and diminishes release of free Ca2+ from the sarcoplasmic reticulum, increasing the likelihood of arrhythmia, which is the most common cause of death.

The decrease in inotropy, contractility and alpha-1 blockade can also cause life-threatening hypotension.

292
Q

Intercondylar origins of the ACL and PCL

A
ACL = anterior
PCL = posterior
293
Q

Pathophysiology of familial pulmonary hypertension

A

Mutation in pro-apoptotic BMPR2 gene leads to proliferation of tunica media, intimal fibrosis and lumenal narrowing.

294
Q

Treatment of pulmonary arterial hypertension while awaiting lung transplant

A

Bosentan, an endothelin receptor antagonist that prevents endothelin-mediated vasconstriction and proliferation of endothelial cells.

295
Q

Opioid receptor that causes miosis

A

Kappa

296
Q

Drugs for Alzheimer’s disease

A

Cholinesterase inhibitors: donepezil
Antioxidants: vitamin E
NMDA receptor antagonists: memantine

297
Q

Pathophysiology of syphilitic aortic aneurisms

A

Vasa vasorum endarteritis and obliteration, leading to weakness of the adventitia.

298
Q

Most common benign tumor of the liver’s histology. When to biopsy

A

Cavernous hemangioma. Cavernous, blood-filled vascular spaces of variable size that are lined by a single epithelial layer. ***Don’t biopsy due to risk of hemorrhage.

299
Q

Mechanism of damage to microvasculature in diabetics

A

Non-enzymatic glycosylation of intima -> vessel hyalinization and reduced blood supply
Glucose accumulation in Schwann cells -> sorbitol conversion by aldose reductase -> cellular swelling and damage.

300
Q

Considerations when combining lipid lowering drugs

A

Gemfibrozil increases serum levels of statins and increases risk of myopathy. Fenofibrate carries a separate risk of myopathy. Bile sequestrants decrease absorption of statins and should be dosed > 4 hours apart.

301
Q

PTSD first line treatment

A

SSRI + CBT

302
Q

Kinesin

A

Microtubule motor protein involved in anterograde transport of synaptic vessicles

303
Q

Why are infarcts rare in patients with PE?

A

Collateral circulation from the bronchial arteries

304
Q

Primary entry site of cryptococcus neoformans. Why does it like to go to the CNS?

A

Primary site is the lungs from inhalation of bird droppings in the soil. The CNS does not have components of the alternate complement pathway necessary for keeping the bug in check, so it likes to go there to proliferate.

305
Q

Where in the heart is the cardiac action potential conducted the fastest and the slowest?

A

“Park At Venture Avenue”

Fastest
-Purkinje System (2m/sec)
-Atrial muscle
-Ventricular muscle
-AV node (0.5m/sec)
Slowest
306
Q

Pathogenesis of migraine headaches

A

Trigeminal afferents that innervate the meninges get activated and secrete substance P and CGRP that cause vasodilation, protein extravasation and neurogenic inflammation. Central nociceptive pathways become more sensitive, explaining the photosensitivity, etc.

Note that triptans block 5-HTB/D receptors, preventing release of vasoactive peptides and causing vasoconstriction.

307
Q

Foscarnet mechanism of action

A

Pyrophosphate analogue that does not require intracellular activation to inhibit herpes DNA polymerase or HIV reverse transcriptase. It is good for treating resistant strains of HSV and CMV.

308
Q

Sofosbuvir mechanism of action

A

Inhibits non-structural protein 5B (NS5B), an RNA-dependent RNA polymerase needed for HCV replication.

309
Q

What fascial planes form the retropharyngeal space?

A

The retropharyngeal space is between the prevertebral fascia and the buccopharyngeal fascia (extends from the carotid sheath and invests the pharyngeal constrictors).

310
Q

Ways to make the diagnosis of cryptococcal meningitis

A

India ink stain, latex agglutination, culture in Saboraud’s agar or methenamine mucicarmine stain

311
Q

What type of virus is HBV?

A

Enveloped partially double-stranded circular DNA with an RNA and DNA-dependent DNA polymerase that

1) Repairs the partial strand to a full strand upon entry to the host cell nucleus
2) Makes mRNA that serves as a template to re-form the partial double stranded DNA again once the new capsid is formed

312
Q

Posterior inferior cerebellar artery occlusion causes…

A

Lateral medullary (Wallenberg) syndrome: contralateral loss of pain/temperature sensation, ipsilateral CN V, VIII, IX, X and XI defects and Horner’s syndrome.

313
Q

Glaucoma drugs

A

Increase outflow by stimulating cholinergic receptors and promoting contraction of the iris sphincter and ciliary muscle, making the lens more convex

  • Prostaglandins: latanoprost, travoprost
  • Cholinergic agonists: pilocarpine, carbachol

Decrease production

  • Carbonic anhydrase inhibitors: topical dorzolamide, acetazolamide
  • Beta-blockers: timolol, betaxolol

Both
-Alpha agonists: brimonidine

314
Q

Fenoldopam mechanism of action

A

Selective D1 agonist used for severe hypertension, especially in patients with renal failure because it is the only agent that increases renal blood flow without increasing blood pressure (dopamine has alpha and beta activity)

315
Q

Petechiae, purpura and ecchymosis

A

Petechiae 1cm

316
Q

A boy has kyphoscoliosis, a high plantar arch and a brother who died of heart disease at age 25. He can no longer play basketball due to discoordination. What is causing his symptoms?

A

Friedrich-Ataxia is an AR mutation in the frataxin gene that codes for assembly of iron-sulfur enzymes. GAA repeats render these enzymes non-functional and there is poor mitochondrial function, oxidative stress and neuronal degeneration in the dorsal root ganglion, posterior column, spinocerebellar and spinothalamic tracts, kyphoscoliosis and hypertrophic cardiomyopathy.

317
Q

AD disorder resulting in peripheral demyelination

A

Charcot-Marie-Tooth disease

318
Q

Why is tocilizumab effective in treating patients with giant cell arteritis?

A

Excess IL-6 drives infiltration of macrophages and PMNs. Tocilizumab is a monoclonal antibody against IL-6 and prevents this cascade from occuring.

319
Q

Important mechanisms by which bugs gain resistance to aminoglycoside antibiotics

A

Methylation of the 30S ribosomal subunit, efflux pumps and enzymes that alter drug structure

320
Q

Why does HDV need HBV for infection?

A

It cannot penetrate host cells unless it is coated by the the HBsAg.

321
Q

Gastric ulcers vs. erosions

A

Ulcers penetrate to the submucosa

Erosions only go as far as the muscularis mucosa, but not through it

322
Q

Where do B-cells develop antibodies specific for the antigens they encounter

A

1) in the bone marrow, Ig undergoes DNA VDJ recombination of the heavy chains and VJ recombination of the light chain
2) affinity maturation occurs in the lymph nodes via somatic hypermutation

323
Q

Part of the retina with greatest visual acuity

A

Macular fovea: in this region each cone synapses to a single bipolar nerve, which synapses to a single ganglion cell.

324
Q

Antibiotics that require protein-mediated transport into bacterial cells

A

Macrolides, fluoroquinolones and aminoglycosides…this is why it can be beneficial to combine these with cell wall synthesis inhibitors.

325
Q

In fetal cardiac development, what is the consequence of:
A) Anterior and cephald deviation of the infundibular septum
B) Abnormal leftward looping of the primitive heart

A

A) tetrology of Fallot

B) transposition

326
Q

Why does squatting help relieve tet spells

A

Increased afterload decreases R -> L shunting from RVOT obstruction.

327
Q

Diffusely hypercellulary glomeruli, granular immunofluorescence and and subepithelial humps on EM

A

PSGN: hypercellularity due to PMN infiltrate, granular immunofluorescence due to C3, IgG and IgM deposition and supepithelial humps are antigen-antibody complexes.

328
Q

Virulence factors of the non-lactose fermenting motile oxidase-negative bacilli that make H2S seen in patients with sickle cell disease.

A

Salmonella has a special capsule with a Vi antigen that prevents it from getting opsinized. This is especially virulent in patients with sickle cell disease due to splenic infarction and susceptibility to encapsulated organisms.

329
Q

Round pearly mass directly behind the tympanic membrane that causes otorrhea and hearing loss.

A

Cholesteatomas. These form from chronic negative middle ear pressure that causes the TM to form retraction pockets that become cystic. The squamous cell debris coalesces to form a cholesteatoma. This mass produces enzymes that can destroy middle ear structures.

330
Q

Why is HDL good

A

It delivers peripheral cholesterol back to the liver via scavenger receptors (SCARB1)

331
Q

Key intracranial landmark for locating the trigeminal nerve

A

The middle cerebellar peduncle extending from the pons

332
Q

What cranial nerve nuclei are located in the:
A) lateral midpons (motor)
B) dorsolateral aspect of the caudal pons with the nerve exiting at the ventrolateral pontomedullary junction
C) floor of the 4th ventricle at the base of the medulla with the nerve exiting at the rostral end of the preolivary sulcus
D) rostral midbrain at the level of the superior colliculus and red nucleus with the nerve exiting the interpeduncular fossa
E) caudal midbrain with the nerve exiting the dorsal midbrain just inferior to the inferior colliculus

A
A) CN V
B) CN VII
C) CN IX
D) CN III
E) CN IV
333
Q

Only cranial nerve to decussate before entering its target muscle

A

CN IV

334
Q

Where in the brainstem would disruption of the corticopontine fibers cause contralateral dysmetria and dysdiadokinesis?

A

The pontocerebellar fibers decussate through the middle cerebellar peduncle into the pons.

335
Q

How is listeria able to live intracellularly?

A

It has listeriolysin O, which forms pores in acidified phagolysosomes that release it into the cytosol.

336
Q

Anterior ECG leads?
Lateral?
Inferior?

A

Anterior = V1-V4 (LAD occlusion leading to anterior wall and anterior septal infarct)
Lateral = I, aVL, V5 and V6 (LCX occlusion)
Note that left main occlusion would result in ST elevations in V1-V6, lead I and aVL
Inferior = II, III and aVF (RCA)

337
Q

Why give second-generation antihistamines to elderly patients with an allergic rash instead of first-generation?

A

Second generation antihistamines block the H1 receptor, reducing mast cell degranulation and improving symptoms. However, they do not block muscarinic (urinary retention, dry mouth), serotonergic (increased appetite) or alpha adrenergic receptors (orthostasis) like the first generation antihistamines do.

338
Q

Phase of cardiac myocyte action potential that correlates with the QRS complex

A

Phase I, this is where class I anti-arrhythmics act

339
Q

How does coronary revascularization resolve a previously hypokinetic region of myocardium

A

Cardiac myocytes undergo hibernation to match the decrease in coronary circulation to prevent necrosis. They are decreasingly responsive to Ca2+, adrenergic stimulation and decrease contractile protein synthesis which makes them hypokinetic. This resolves with restoration of coronary blood flow.

This is different from ischemic preconditioning where repeated episodes of angina can actually delay myocyte death upon complete arterial occlusion.

340
Q

Difference between zero order and first order kinetics

A
Zero-order = same AMOUNT is always metabolized, metabolism curve is flat and elimination curve is linearly negative
First-order = same PERCENT is always metabolized, metabolism curve is linearly positive, elimination curve exponentially decreases
341
Q

Nutrient deficiencies seen following gastric bypass with removal of the duodenum.

A

Fe, B12 and the ADEK vitamins (especially vitamin D)

342
Q

Condition with reduced compound muscle action potential (CMAP) that can be overcome with repetitive high frequency nerve stimulation

A

Botulism

343
Q

A) Jervell and Lange-Nielson syndrome

B) Romano Ward syndrome

A

A) AR Congenital long QT + sensorineural hearing loss

B) AD congenital long QT with normal hearing

344
Q

Ear canal sensory innervation

A

Mostly from V3 (auriculotemporal branch) but posterior part is innervated by CN X auricular branch.

345
Q

Cancer risk when doing an axillary lymph node dissection in women with breast cancer.

A

This can predispose them to chronic lymphedema and lymphangiosarcoma.

346
Q

Antibodies against desmoglein 1 and 3

A

Pemphigus vulgaris

347
Q

Pathophysiology of the osteolytic bone lesions seen in multiple myeloma

A

Plasma cell secretion of IL-1 and IL-6 results in osteoclast activation

348
Q

Why is PaO2 slightly decreased in the left atrium compared to the pulmonary capillary bed?

A

The bronchial arterioles and small cardiac (thesbian) veins empty into the pulmonary veins, which empty into the LA

349
Q

Causes of hereditary pancreatitis

A

Trypsin that is resistant to inactivating cleavage by trypsin and SPINK1 peptidase inhibitor that reduces inhibitory effects on trypsinogen activation.

350
Q

How does hepcidin reduce serum iron levels

A

It inhibits iron transport across ferroportin-1 in the gut mucosal cells and in the circulating macrophages.

351
Q

Mechanism of action of “RIPE” drugs for Tb and side effects to look out for

A

Rifamycin: inhibits bacterial DNA-dependent RNA polymerase, red fluids and cytopenia
Isoniazid: inhibits mycolic acid synthesis after activation by mycobacterial catalase-peroxidase, neuropathy corrected by B6 supplementation, hepatotoxic
Pyrazinamide: mechanism unclear, hepatotoxic
Ethambutol: inhibits arabinosyl transferase activity in peptidoglycan wall synthesis, optic neuropathy

352
Q

Why are fibrates a risk factor for development of cholesterol gallstones?

A

They decrease enzymatic activity of 7alpha-hydroxylase, reducing conversion of cholesterol to bile acids

353
Q

How does cardiac disease cause pulmonary hypertension

A

Increased pulmonary venous pressure causes decreasedd NO release and increased endothelin release. Endothelin causes vasoconstriction, medial hypertrophy, intimal thickening and fibrosis, resulting in increased resistance and hypertension.

354
Q

What triggers HCO3- release into the small intestine?

A

Tactile stimulation, increased parasympathetic activity and acid-induced secretin release all trigger HCO3- release from Brunner glands in the duodenal mucosa and from the pancreas.

355
Q

Patient has fungus with:
A) Branching hyphae with culture @ 25C, cigar-shaped budding yeast @ 37C with bx
B) Hyphae @ cx, thick-walled spherules filled with endospores @ bx
C) Branching hyphae @ cx, oval yeast in macrophages @ bx
D) Branching hyphae @ cx, large round yeast with doubly refractile wall and broad-based budding at bx
E) Blastoconida @ cx, cells covered in budding blastoconida @ bx

A
A) sporothrix
B) coccidioides
C) histoplasma
D) blastomyces
E) paracoccidioides
356
Q

3 phases of stomach acid stimulation in the stomach

A

1) Cephalic: thought, smells -> parasympathetic stimulation of ECL cells and acid release
2) Gastric: presence of food in stomach -> gastrin-mediated stimulation of ECL cells and acid release
3) Intestinal: food moves into intestine and release of peptide YY, somatostatin and secretin decrease gastrin release and stimulation of ECL cells

357
Q

18-month old child with X-linked eczema, thrombocytopenia and recurrent infections from encapsulated organisms and opportunistic organisms.

A

Wiskott-Aldrich -> B & T-cell deficiency resulting in recurrent encapsulated and opportunistic infections after maternal IgG goes away.

358
Q

Child with oculocutaneous albinism, peripheral neuropathy and recurrent infections due to poor fusion of phago-lysosome. Path findings?

A

Chediak-Higashi syndrome. Light microscopy reveals giant lysosomal inclusions due to the defective phagolysosome fusion.

359
Q

Where in the foot would you have a lesion that presents with only superficial inguinal lymphadenopathy vs. concominant popliteal lymphadenopathy?

A

Lateral foot = popliteal + inguinal

Medial foot = inguinal only

360
Q

Foamy macrophages with PAS + material, rod shaped intracellular bacteria and diastase-resistant particles.

A

T. whippelii

361
Q

Most common anomalies associated with imperforate anus

A

Genital tract abnormalities

362
Q

How does IgM stimulate phagocytosis compared to IgG

A

IgM recruits C3b, where phagocytes have actual receptors for the Fc region of IgG.

363
Q

How is CO2 carried from the periphery back to the lungs for expiration?

A

It is taken up by RBCs, turned into HCO3- and released in exchange for Cl-. It is also carried on Hgb attached to an NH.

364
Q

Murmurs that decrease with handgrip

A

This increases after load only, so AS and HOCM get better.

365
Q

Childhood tumor with paraneoplastic syndrome of opsoclonus, myoclonus and truncal ataxia

A

Neuroblastoma in adrenal medulla due to n-myc mutation. Note that the prognosis is good if the child is

366
Q

Granulomatosis with polyangiitis antibodies

A

c-ANCA against neutrophil proteinase-3

367
Q

SMA syndrome

A

Decreased angle of the SMA that causes entrapment of the transverse duodenum between the SMA and the aorta. Note also that these patient may present with varicocele due to compression of the left renal vein at this point.

368
Q

Anti arrhythmic with side effect of:
A) Blue-grey skin discoloration, pulmonary fibrosis and thyroid dysregulation
B) Drug-induced lupus
C) Constipation + gingival hyperplasia
D) Delirium + GI sx + changes in color vision

A

A) Amiodarone
B) Procainamide
C) Verapamil
D) Digoxin

369
Q

Why is the IMA sometimes not reconnected after colon surgery?

A

The SMA anastomoses with the IMA at the marginal artery of Drummond and the arc of Riolan

370
Q

HEV viral features

A

Unenveloped ssRNA that causes fulminant hepatitis in pregnant women

371
Q

Pathophysiology of normal pressure hydrocephalus

A

Stretching of the sacral motor fibers = ataxia + incontinence
Periventricular limbic system distention = dementia

372
Q

Adolescent presents with brief myoclonic seizures in both upper extremities and does not lose consciousness during episodes. Episodes occur more frequently when waking and when sleep deprived. How should you treat him?

A

Juvenile myoclonic epilepsy is a form of generalized epilepsy that merits treatment with broad-spectrum anti seizure drugs such as lamotrigine, valproate, levatiracetam and topiramate.

373
Q

Narrow spectrum anti seizure medications that are used for focal seizures and not generalized epilepsy

A

Carbamazepine, gabapentin, phenobarbital and phenytoin

374
Q

Baclofen mechanism of action

A

GABA-B agonist

375
Q

What about the inflammatory state increases the ESR

A

During inflammation, IL-1, IL-6 and TNF-alpha stimulate hepatic production of acute phase proteins like fibrinogen. Fibrinogen causes rapid erythrocyte stacking into Reuleaux formations and increases ESR.

376
Q

Most likely condition in a patient with a febrile viral illness involving a morbilliform rash, encephalitis and flaccid paralysis.

A

West nile virus, a +ssRNA transmitted by the Culex mosquito in the summer time.

377
Q

Traztuzumab mechanism of action

A

Monoclonal antibody that binds to the extracellular domain of Her2 and prevents activation of the transmembrane tyrosine kinase. This down regulates proliferation and promotes neoplastic cell apoptosis.

378
Q

Denosumab mechanism of action

A

Monoclonal antibody against RANKL that inhibits osteoclast activation and bone resorption

379
Q

Antibiotics effective against anaerobes

A

Metronidazole, clindamycin, carbapenems and piperacillin-tazobactam (extended spectrum PCN + beta-lactamase inhibitor)

380
Q

Antibiotics not susceptible to beta-lactamase, therefore organisms only develop resistance to these via alterations in their penicillin-binding proteins.

A

Cephalosporins, carbapenems and penicillinase-resistant penicillins (methicillin, nafcillin)

381
Q

Patients with abnormal DHR flow cytometry testing are at increased risk for infection from what types of bugs?

A

Chronic granulomatous disease is a result of impaired intracellular killing of bacteria and fungi due to deficiency in NADPH oxidase. This enzyme is key in production of superoxide (O2-) that can be further converted to hydrogen peroxide and hydroxyl radicals needed for intracellular killing of pathogens. Despite absence of the enzyme H2O2 can build up from other sources and kill organisms that are catalase negative; however, patients are at high risk for infection by catalase positive organisms like S. aureus, nocardia, aspergillus, burkhoderia and serrate because they can break down the H2O2.

382
Q

Liver biopsy in a child with Reye’s syndrome

A

Micro vesicular steatosis, cellular swelling, decreased mitochondria and glycogen. Liver damage leads to hepatic encephalopathy in these patient.s

383
Q

2 phases of HBV infection

A

Proliferative: all antigens are expressed on hepatocyte surface, MHC I is up regulated and cells are being killed by cytotoxic T-cells

Integrative: antigens are reduced on cell surface and there is reduced cytotoxic T-cell killing; however, HBV DNA is incorporated into host DNA and there is continued risk for HCC due to production of HBx protein that inactivates p53

384
Q

Pathophysiology of low dose aspirin for stroke prevention

A

Low dose preferentially inhibits TxA2 production, maintaining production of PGI2 (prostacyclin) by endothelial cells that prevents platelet aggregation.

385
Q

Most significant risk factor for aortic dissection

A

HTN

386
Q

RBC inclusions seen with sideroblastic anemia

A

Pappenheimer bodies (iron). These are different from the Howell-Jolly bodies seen in splenectomized patients (DNA)

387
Q

How does urea contribute to urine concentration

A

When ADH is release to concentrate the urine, it allows passive diffusion of urea across the medullary collecting duct to maximize the osmolarity of the interstitium to augment H2O reabsorption from the collecting duct.

388
Q

How does HBV infection predispose patients to HCC?

A

It integrates into host DNA, suppresses p53 and up regulates IGF-2 production and IGF-1 receptors.

389
Q

Clinical manifestation when there are portocaval anastomoses between:
A) Left gastric vein and esophageal vein
B) Superior rectal vein and middle/inferior rectal veins
C) Paraumbilical veins and superficial/inferior epigastric veins

A

A) Esophageal varices
B) Hemorrhoids
C) Caput medusae

390
Q

Cause of congenital jejunal atresia

A

Vascular occlusion (as opposed to failure to recannalize w/duodenal atresia). Kids will have a jejunum that ends in a blind loop and an “apple core” ileum spiraled around an ileocolic vessel

391
Q

Train of four stimulation in a patient on succinylcholine vs. rocuronium

A

Succinylcholine is a depolarizing NMJ blocker, so there is an initial uniform decrease in stimulation. If exposure to succinylcholine persists, or the patient is a slow metabolizer, the train of four transitions to a stepwise decrease in stimulation similar to non-depolarizers.

Rocuronium is a non depolarizing NMJ competitive antagonist, so there is a progressive stepwise decline in stimulation with each electrical signal fire.

392
Q

Drugs that prevent peripheral degradation of levodopa so more of it can get to the brain

A

Carbidopa - inhibits dopa decarboxylase

“capone” drugs - inhibits COMT methylation of levodopa

393
Q

Toxin responsible for the pseudomembranes seen in C. difficile colitis

A

B: causes cell necrosis and fibrin deposition. Toxin A is responsible for the watery diarrhea.

394
Q

Kidney stones that need urinary alkalization for treatment

A

Cystine and uric acid stones.

395
Q

Sodium-cyanide nitroprusside test

A

Looks for cystine in the urine to diagnose patients with autosomal recessive mutation in the COLA (cystine, ornithine, lysine and arginine) dibasic amino acid transporter in the PCT.

396
Q

How to differentiate E. coli from enterobacter

A

They are both gram-negative, fast lactose fermenting rods; however, E. coli can ferment tryptophan to indole and enterobacter cannot.

397
Q

Great medication for insomnia in the elderly that cannot tolerate anti-histamines or benzodiazepines

A

Ramelteon, a melatonin agonist

398
Q

Lumbar plexus nerve that innervates:
A) suprapubic region and abdominal muscles
B) upper thigh, inner leg and motor extensors of leg
C) upper anterior thigh and cremaster
D) upper and medial thigh, parts of genitalia, runs with spermatic cord
E) lateral thigh
F) medial thigh and adductors

A
A) Iliohypogastric
B) femoral
C) genitofemoral
D) ilioinguinal
E) lateral femoral cutaneous nerve
F) obturator
399
Q

Capsacian cream mechanism of action

A

Substance P depletion

400
Q

Subtypes covered in the quadrivalent meningococcal vaccine

A

A, C, W-135 and Y

401
Q

Enzyme specific to mast cells that can be used to support the diagnosis of anaphylaxis

A

Tryptase

402
Q

How does acetaminophen reduce fevers in children?

A

Decreased levels of PGE2 -> reduced hypothalamic set point -> reduced fever

403
Q

Atropine is contraindicated in patients with what eye condition?

A

Glaucoma. The anti-cholinergic effect causes mydriasis which can precipitate acute angle glaucoma

404
Q

Only branch of the trigeminal nerve that has motor innervation

A

Mandibular branch

405
Q

A patient presents immediately after surgery with fevers, tenderness to palpation of the RUQ, extreme tranaminitis and prolonged PT. She expires and autopsy shows a shrunken liver with centrilobular necrosis and inflammation around the portal tracts. What caused her condition?

A

Inhaled-anesthetic hepatotoxicity…most likely from halothane.

406
Q

Clinical disinfectant that works by:
A) disrupting cell membranes, denaturing proteins, but does not affect spores
B) disrupting cell membranes, coagulating the cytoplasms, but does not affect spores
C) produces destructive free radicals that oxidize cellular components and kills spores
D) halogenates proteins and nucleic acids and kills spores

A

A) alcohol
B) chlorhexidine
C) hydrogen peroxide
D) iodine

407
Q

Aminoglycoside mechanism of action

A

Covalently bind 30S subunit, altering mRNA reading and impacting translocation of tRNAs.

408
Q

Progressively worsening dizziness, dysarthria, visual disturbances, limb and truncal ataxia in a patient with small cell lung cancer, breast, ovarian or uterine cancer.

A

Paraneoplastic subacute cerebellar degeneration secondary to development of anti-Hu, anti-Yo and anti-P/Q antibodies that attack the Purkinje cells of the cerebellum.

409
Q

Main difference between unfractioned heparin and LMWH?

A

Unfractioned heparin binds ATIII which inhibits Xa AND thrombin.
LMWH binds ATIII and really only inhibits Xa

410
Q

Direct thrombin inhibitors

A

Argatroban, lepirudin and dabigatran

411
Q

Platelet ADP-receptor inhibitors

A

Clopidogrel and ticlopidine

412
Q

Why is 30 minutes the cutoff for reversible myocardial ischemia after an MI?

A

During the 1st 30 minutes ATP stores are being depleted to ADP, AMP and adenosine. Adenosine acts as a local vasodilator for the poorly perfused myocardium, but typically runs out by 30 minutes of ischemia.

413
Q

A patient presents with a skin lesion. Biopsy shows PMN clusters in the superficial dermis and parakeratotic stratum corneum. There are also plaques with scales that have pinpoint bleeding when the scale is removed. What does this patient have?

A

Psoriasis is characterized by Munro abscesses (PMN collections in superficial dermis and parakeratotic region) and Auspitz sign (pinpoint bleeding when scale is removed)

414
Q

Auscultatory findings that can help determine the severity of mitral stenosis

A

A2-OS interval. As left atrial pressure increases from mitral stenosis, the opening snap will be heard closer and closer to A2 because the increasing atrial pressure overcomes the ventricular pressure at a higher level.

415
Q

Black colonies on cysteine-tellurite agar that demonstrate intracytoplasmic metachromatic granules after growth on Loffler’s medium and staining with aniline dye.

A

C. diphtheriae

416
Q

Thayer-Martin media

A

Neisseria

417
Q

Function of internal branch of the superior laryngeal nerve

A

Sensory or autonomic innervation. Main clinical significance is afferent limb of cough reflex as it lies in the piriform recess where food passes beyond the epiglottis.

418
Q

Most common cause of death in a patient hospitalized for MI

A

Cardiogenic shock from LV failure

419
Q

Absolute contraindications for OCPs

A
Prior thromboembolic event
Hx of estrogen-dependent tumor
Women > 35 who smoke
Hypertriglyceridemia
Pregnancy
Acute or decompensated liver failure
420
Q

Drug you can use for post-operative urinary retention

A

Bethanechol is a muscarinic agonist that improves bladder emptying in these patients. Note this is in contrast to oxybutinin, which is an anti-muscarinic used for patients with detrusor overactivity leading to urge incontinence.

421
Q

Symptoms of vitamin E deficiency

A

It closely mimics Friedrich ataxia and vitamin B12 deficiency (without the megaloblasts) due to degeneration of the spinocerebellar and dorsal columns.

422
Q

What type of patients may be genetically predisposed to infection by giardia?

A

Those resulting in IgA deficiency (X-linked Bruton’s agammaglobulinemia, SCID and common variable immune deficiency) because IgA is necessary to prevent giardia from latching on to the GI mucosal walls and for opsonization.

423
Q

Isoproterenol mechanism of action

A

Beta-1 and Beta-2 agonist

424
Q

Cardioselective vs. non-selective beta-blockers

A

A -> M are cardioselective

N -> Z are non-selective

425
Q

Factor Xa inhibitors

A

RivaroXAban, apiXAban, fondaparinux (indirect)

426
Q

GpIIb/IIIa receptor blocker

A

Eptifibatide

427
Q

ADP P2Y12 receptor blocker

A

Ticagrelor, prevents activation of GpIIb/IIIa activating complex

428
Q

Anti-arrhythmics that can cause QT prolongation

A

Class Ia and III

429
Q

Most common cause of malignant acanthosis nigricans

A

Malignant gastric adenocarcinoma

430
Q

How to detect deficiencies in aldolase B or fructokinase

A

Copper reduction test, presence of reducing sugars like fructose-1P and fructose will result in a positive test

431
Q

Patient with periportal pipestem fibrosis

A

Pathognomonic for schistosomiasis because the parasites mature in the liver and elicit a Th2-mediated immune response resulting in infiltrating Th2 cells, eosinophils and M2 macrophages. Long-term infection results in significant fibrosis.

432
Q

Strong predisposing anatomic abnormality for Mallory-Weis tears?

A

Hiatal hernia

433
Q

Hereditary cutaneous facial angiomas in a V1 and V2 distribution and leptomeningeal angiomas with retardation, tram-track skull radiographic opacities and seizures.

A

Sturge-Weber

434
Q

Kussmauls sign

A

Paradoxical increase in JVD w/inspiration due to constrictive pericarditis or tamponade.

435
Q

TTP pathophysiology

A

Decreased ADAMTS13 enzymatic cleavage of vWF multimers results in diffuse microvascular platelet-rich thrombi, hemolytic anemia and thrombocytopenia with associated fever, neurologic symptoms and renal failure.

436
Q

Pathogenesis of cholecystitis

A

Outflow obstruction -> lysolethicin activation -> gallbladder wall irritation and PG release -> hypo motility and distention -> ischemia -> bacterial colonization

437
Q

What happens to GFR, RPF and filtration fraction in hypovolemia?

A

RPF decreases due to decreased renal blood flow
Due to RAAS activation, GFR decreases less than RPF
Since FF = GFR/RPF, FF increases

438
Q

How to determine renal plasma flow

A

RBF * (1 - Hct)

439
Q

Measurements of osteoblast and osteoclast activity in bone

A
Osteoblast = bone specific ALP (denatures with boiling)
Osteoclasts = urinary deoxypyrinidoline
440
Q

Molecular make up of H. influenzae type B’s capsule

A

Polyribosyl-ribitol-phosphate

441
Q

Lumbrical function

A

Lumbricals: MCP flexion, PIP extension

442
Q

Why is development of a vaccine of N. gonorrhoeae difficult

A

The proteins that make up the pili undergo antigenic variation

443
Q

HIV mutation that renders the virus resistant to standard protease inhibitors, NRTIs and NNRTIs.

HIV mutation that allows the virus to escape host humoral immunity

A

Pol gene mutations result in variations in HIV proteases and alteration of the structure of the HIV reverse transcriptase.

Env gene mutations result in variations in structural glycoproteins (gp 120 and gp 41) that allow it to evade neutralizing antibodies

444
Q

Causes of myasthenia gravis exacerbations in patients being treated with an AChE inhibitor like neostigmine

A

1) Inadequate dose of AChE inhibitor, demonstrated by symptoms improvement after edrophonium infusion
2) Cholinergic crisis where there is too much ACh in the synaptic cleft, the AChR becomes resistant to ACh stimulation and patients will have persistent weakness despite edrophonium infusion

445
Q

Pralidoxime

A

Medication used in organophosphate poisoning that regenerates cholinesterase. Useful for nicotinic and muscarinic overstimulation. This is why it is better than atropine (only hits muscarinic receptors).

446
Q

Radial nerve injury from Saturday night palsy vs. being a mechanic

A

Saturday night palsy: wrist drop and forearm sensory loss
Mechanic: impingement at supinator canal after ECRL and sensory branches have been given off, so only finger drop (impaired extension of thumb and index finger) with preserved sensation.

447
Q

t(8;14)

A

Burkitt lymphoma, due to activation of the c-myc oncogene which acts as a transcription activator and promotes cellular proliferation

448
Q

Why is the use of succinylcholine in burn, crush injury or neurodegenerative disorder patients contraindicated?

A

These patients have unregulated AChR due to decreased stimulation or systemic inflammation. Giving succinylcholine to these patients results in massive release of K+ and life-threatening arrhythmias.

449
Q

Probenecid for infection mechanism of action

A

Decreases renal clearance of penicillins by inhibiting the tubular organic acid secreting system

450
Q

AV nodal blood supply in a left vs. right dominant coronary circulation

A

Left dominant = LCX -> posterior descending artery -> AV nodal artery
Right dominant = RCA -> posterior descending artery -> AV nodal artery

451
Q

How does chronic NSAID use result in chronic interstitial nephritis?

A

NSAIDs concentrate in the medulla, they uncouple oxidative dephosphorylation and result in glutathione depletion. This causes lipid peroxidation, tubular damage, necrosis, atrophy, scarring and calcification.

NSAIDs also decrease prostaglandin synthesis, which may result in vasoconstriction of the medullary vasa recta and ischemic papillary necrosis.

452
Q

Size of colonic adenomas at high risk for malignancy

A

> 4cm

453
Q

Mutations that cause specific changes in the adenoma carcinoma sequence

A

1) APC inactivation in normal colonic mucosa
2) Methylation abnormalities and COX-2 over expression -> hyper proliferating mucosa
3) K-RAS mutation leads to unregulated proliferation
4) DCC mutation leads to avoidance of apoptosis
5) p53 mutation leads to carcinoma

454
Q

FAP vs. Lynch syndrome mutations

A
FAP = APC and presents as classic adenoma
Lynch = microsatellite instability due to mutation in mismatch repair genes (MLH1, MSH2, PMS2 and MSH6) and presents with macroscopical normal dysplastic epithelium
455
Q

Why are dendritic cells the most effective type of antigen presenting cells?

A

They constitutively express MHC II + B7 that can bind to T-cell CD28

456
Q

Pathophysiology of functional hypothalamic amenorrhea

A

Decreased adipose -> decreased leptin -> decreased GnRH

457
Q

Why is surfactant so freaking awesome.

A

According to the law of LaPlace: pressure = (surface tension x 2)/radius. This means that smaller alveoli have a greater tendency to collapse and larger alveoli have a greater tendency to expand. When you add surfactant to the alveoli, it reduces the surface tension as it gets packed closer together in the smaller alveoli; however, it has minimal effect as it gets spread out in the larger alveoli….keeping large and small alveoli in check.

458
Q

A patient presents with myalgia, proximal muscle weakness, cramping and myoedema, labs show an elevated CK.

A

Myoedema is a characteristic of hypothyroid myopathy due to slow reabsorption of Ca2+ into the sarcoplasmic reticulum.

459
Q

Skin lesion with:
A) small cells resembling basal cells with variable pigmentation, hyperkeratosis and keratin cysts
B) nests of uniform round cells in the basal epidermis and upper dermis with inconspicuous nuclei and few mitotic figures
C) nests of uniform round cells at the dermal-epidermal junction with inconspicuous nuclei and few mitotic figures
D) nests of uniform round cells in the dermis with inconspicuous nuclei and few mitotic figures taken from a non-pigmented skin lesion
E) large cells with prominent nucleoli and atypical mitotic activity in the superficial epidermis
F) mildly atypical cells that form nests and coalesce into a continuous layer at the dermal-epidermal junction

A
A) seborrheic keratosis
B) compound nevus
C) junctional nevus
D) intradermal nevus
E) superficial spreading malignant melanoma
F) atypical (dysplastic) nevus
460
Q

Anti arrhythmic with highest selectivity for ischemic myocardium

A

Lidocaine, it has a high selectivity for rapidly depolarizing or depolarized cells (ischemic myocardium)

461
Q

IGF-1 deficiency vs. FGF-3 mutation

A
IGF-1 = proportional decrease in growth
FGF-3 = normal axial spine length, but short long bones due to poor growth at epiphyseal plates.
462
Q

Organophosphate poisoning symptoms

A
Diarrhea + diaphoresis
Urination
Miosis
Bronchospasm, bronchorrhea, bradycardia
Emesis
Lacrimation
Salivation
463
Q

Give FFP for heparin overdose?

A

No! It contains AT III and will enhance the effects of heparin.

464
Q

Components of cryoprecipitate

A

Factors VIII, XIII, fibrinogen and vWF

465
Q

1st muscle penetrated when placing a chest tube in the mid-axillary line

A

Serratous anterior if above 5th rib, external oblique if below 5th rib

466
Q

Pathogenesis of alcohol related hepatic steatosis

A

Increased NADH production by alcohol dehydrogenase and aldehyde dehydrogenase results in decreased beta-oxidation and triglyceride accumulation within hepatocytes.

467
Q

t(11;14)

A

Mantle cell lymphoma due to translocation of the cyclin D1 locus to a region close to the Ig heavy chain on chromosome 14. This results in increased G1 -> S phase transition.

468
Q

Why does lung parenchyma get destroyed in patients with pulmonary abscesses?

A

Release of lysosomal enzymes by macrophages and PMNs.

469
Q

Dubin-Johnson syndrome

A

AR mutation in MRP2 that limits conjugated bili excretion into bile canaliculi -> asymptomatic + dark liver

470
Q

Symptoms of central pontine myelinolysis

A

Spastic quadriplegia and pseudo bulbar palsy of cranial nerves IX, X and XI

471
Q

t(15;17)

A

Alpha retinoic acid receptor on chromosome 17 is translocated to PML gene on chromosome 15. This results in promyelocytes that cannot differentiate and APML (M3 AML) that can be treated with all-trans retinoic acid that binds the retinoic acid receptor and stimulates promyelocyte differentiation.

472
Q

Why are vemurafenib and imatinib good drugs for melanoma?

A

Melanocytes become malignant via the KIT receptor kinase -> BRAF pathway. Imatinib inhibits KIT receptor kinase and vemurafenib inhibits the mutant BRAF mutation, preventing cyclin D1-induced transition of malignant melanocytes from G1 to S phase.

473
Q

Pathogenesis of myasthenia gravis

A

Autoantibodies against the post-synaptic AChR -> receptor degradation and endplate damage due to complement fixation -> overall decrease in motor endplate potential

474
Q

Streptomycin mechanism of action

A

Interferes with the 16s rRNA subunit of the 30S ribosomal subunit

475
Q

Enterococci are resistant to which drugs?

A

Aminoglycosides: chemically modify the drug to prevent its interaction with ribosomes
Penicillins: beta-lactamase + altered penicillin binding protein
Vancomycin: substitutes D-alanine D-lactate for the typical D-alanyl D-alanine used to synthesize the cell wall
Trimethoprim-sulfamethoxazole

476
Q

Mechanism of maintenance of cardiac output in patients with chronic vs. acute aortic regurgitation

A
Chronic = eccentric ventricular dilation = increased SV
Acute = increased HR
477
Q

Structures that run around the piriformis

A

Superior: superior gluteal vessels and nerves
Inferiorly: inferior gluteal vessels, internal pudendal vessels and multiple nerves to include the sciatic nerve

478
Q

Why do patients with Bell’s palsy get hyperacousis?

A

Paralysis of the stapedius results in increased sound transmission from the stapedius to the oval window

479
Q

Diphtheria vaccine target

A

Binding subunit (B) of the diphtheria exotoxin

480
Q

Complications associated with uncal herniation

A

1) CN III palsy
2) Contralateral and/or ipsilateral paresis due to compression of the cerebral peduncles in the midbrain
3) homonymous hemianopia with macular sparing due to compression of the posterior cerebral artery

481
Q

Age groups that should receive the PCV13 in addition to the PPSV23

A

> 65 years old, immunosuppressed or

482
Q

Pneumoconiosis with:
A) diaphragmatic pleural plaques
B) enlarged hilar lymph nodes and non-caseating granulomas
C) multiple discrete nodules in the upper lobes
D) nodular densities and hilar eggshell calcifications

A

A) asbestosis
B) berylliosis
C) coal worker’s lung
D) silicosis

483
Q

Anti-convulsant that blocks presynaptic voltage-gated Ca2+ channels

A

Gabapentin

484
Q

Infections and drugs that can trigger pancreatitis

A

Infections: coxsackie and mycoplasma
Drugs: azathioprine, sulfasalazine, valproate and furosemide

485
Q

Histology of the most common benign pulmonary mass

A

Hamartomas consist of hyaline cartilage, fat, smooth muscle and clefts lined by respiratory epithelium. Note that these tumors present as popcorn calcifications on CXR

486
Q

Compensatory mechanism that limits clinically apparent edema in the initial stages of fluid overload

A

Lymphatic drainage

487
Q

Ixodes tick transmits what pathogens

A

Borrelia, anaplasma and babesia

488
Q

Serious complication of babesiosis in splenectomized patients

A

ARDS

489
Q

Calculate volume of distribution

A

drug given / plasma drug concentration

490
Q

Characteristics of a drug with very low volume of distribution (~4)

A

It is highly bound by albumin, has a large molecular weight or is highly charged making it hydrophilic, keeping it within the vascular compartment.

491
Q

Characteristics of a drug with very high volume of distribution (~41)

A

Low molecular weight and uncharged

492
Q

Characteristic of a drug with a volume of distribution around 14L

A

Low molecular weight, but charged (hydrophilic). This allows it to spread to the interstitial fluid, but cannot cross barriers that lipophilic drugs can.

493
Q

Needle shaped clefts in renal arcuate or intralobular arteries

A

Renal atheroembolism, the cholesterol dissolved on tissue preparation

494
Q

Places you find thick, viscous mucus in patients with CF

A

Lungs, pancreas, intestine, biliary tree and salivary glands

495
Q

Nerve that supplies sensation to…
A) Lateral forearm
B) Medial forearm
C) Posterior arm are posterior forearm

A

A) Lateral cutaneous nerve of the forearm, a branch off the musculocutaneous nerve
B) Medial cutaneous nerve of the forearm, a branch off the medial cord of the brachial plexus
C) Radial nerve

496
Q

Differentiating a PFO from ASD

A

ASD has wide fixed splitting of S2 and PFO does not

497
Q

Function of the cells that make up a glomus tumor

A

They are specialized smooth muscle cells involved in temperature sensation and dermal vasoconstriction.

498
Q

Differentiating B from T cell ALL

A

All will be TdT+ because they are lymphocyte precursors

  • B lymphoblasts: CD10, CD19 and CD20 positivity
  • T lymphoblasts: CD2, CD3, CD4, CD5, CD7 and CD8
499
Q

In the sympathetic nervous system, thoracic pre-ganglionic neurons release ACh onto the post-ganglionic neurons. The post-ganglionic neurons then release NE onto their target tissue. What are the two exceptions to this rule?

A

Adrenal medulla, releases NE and EPI after direct pre-ganglionic ACh stimulation.

Eccrine sweat glands of skin, release ACh from the post-ganglionic sympathetic nerve fibers

500
Q

Why do patients with nephrotic syndrome have foamy urine?

A

The urinary albumin loss is so great that the liver cannot compensate with increased synthesis of albumin. At the same time, the liver is cranking out lipoproteins like VLDL, LDL, lipoprotein A and apolipoproteins that increase plasma oncotic pressure. These lipids also spill into the urine and result in lipiduria.

501
Q

Why is there only a modest increase in SVR during exercise when catecholamines cause vasoconstriction in so many organs to shunt blood to the muscles?

A

Exercising muscle produces lactate, K+, adenosine, ATP and CO2 that cause dramatic vasodilation in the muscles. Although there is still an increased SVR, it is not as high as one would expect due to the vasodilatory effects of these molecules.

502
Q

Drug for patients with BPH urinary retention but no hypertension

A

Tamsulosin only blocks the alpha1B receptors in the prostate and leaves the alpha1 receptors in blood vessels alone.

503
Q

Mechanism of action of the drug used to treat oral candidiasis

A

Nystatin works similarly to amphotericin by binding ergosterol and poking holes in the fungal membrane.

504
Q

Staining to identify T. whippelii

A

PAS +, diastase resistant (because diastase breaks down glycogen) in intestinal biopsy

505
Q

Tb infection pathogenesis

A

Inhaled droplets -> phagocytosis -> sulfatide virulence factor promotes intracellular proliferation -> macrophage lysis and infection/lysis of surrounding macrophages -> macrophages eventually carry antigen to T-cells, but this does not happen for 2-4 weeks after infection.

506
Q

Cells that give rise to clear cell renal carcinoma

A

PCT

507
Q

Renal oncocytoma original cell

A

CD

508
Q

A 1 year old child presents with progressive ataxia, retinitis pigmentosa, greasy, foul-smelling stools and a peripheral blood smear demonstrating acanthocytes. Labs reveal complete absence of VLDL. What is the cause of his foul-smelling stools.

A

He has AR abetalipoproteinemia. This disorder is a result of a mutation in the MTP gene that results in loss of function of a protein that assists apoB folding and transfer of lipids to newly formed chylomicrons (apo-B48) and VLDL particles (apoB-100). Inability to transport dietary lipids results in accumulation within enterocytes (foamy cytoplasm) and malabsorption. The acanthocyte and neurologic abnormalities are a result of malabsorption of fat soluble vitamins like vitamin E.

509
Q

These congenital heart defects are associated with:
A) ostium primum ASD, regurgitant AV valves
B) tetrology of Fallot and aortic arch abnormalities
C) HCM
D) aorta cystic medial necrosis
E) cardiac rhabdomyoma valvular obstruction
F) aortic coarctation
G) supravalvular aortic stenosis

A
A) Trisomy 21 (endocardial cushion defects)
B) DiGeorge
C) Friedrich's ataxia
D) Marfan's
E) Tuberous sclerosis
F) Turner's
G) William's
510
Q

How do leukocytes exit the bloodstream

A

1) Rolling - selections
2) Adhesion - integrins ***CD18 deficiency = inability to make integrins -> LAD
3) Transmigration

511
Q

Difference between myeloperoxidase deficiency and chronic granulomatous disease?

A

In chronic granulomatous disease, NADPH oxidase is deficient, however, catalase positive bacterial create their own H2O2, which can then be converted into a hydroxyl radical and be killed as it would have been in the first place.

In myeloperoxidase deficiency, the H2O2 produced cannot be converted to the hydroxyl radical and the patient is unable to kill catalase negative AND catalase positive pathogens.

512
Q

Characteristic bone feature of patients with hyperparathyroidism

A

PTH stimulates resorption of CORTICAL bone resulting in subperiosteal thinning

513
Q

Characteristic bone feature of patient with osteopetrosis

A

Persistence of primary unmineralized spongiosa in the medullary canals.

514
Q

What would happen to muscle contraction if there were not any T-tubules

A

Muscle would not be able to contract all at the same time.

515
Q

Why are infants born before 32 weeks gestation at high risk for inter ventricular hemorrhage.

A

The germinal matrix is still present beneath the ventricles. This is the region where glial cells and neurons migrate out from during development. The vascular area is not well supported by glial cells and therefore the blood vessels are prone to rupture…even more so given the immature cerebral blow flow auto regulation in infants.

516
Q

Pathogenesis of HIV-associated dementia

A

HIV-infected monocytes cross the BBB and become perivascular macrophages -> activated macrophages and microglia form multinucleate giant cell clusters around areas of necrosis in the subcortical regions (microglial nodules) -> HIV-derived proteins and local cytokine release causes neurotoxicity

517
Q

Dementia associated with large areas of encephalomalacia

A

Vascular dementia

518
Q

Why do patients with salmonella get hepatosplenomegaly

A

The bacteria invade via intestinal macrophages, replicate in Peyer’s patches and then spread throughout the reticuloendothelial system.

519
Q

Common causes of non-bacterial thrombotic endocarditis

A

1) Hypercoaguability: mucinous adenocarcinomas of the pancreas and adenocarcinomas of the lung

520
Q

Dysplasia vs. carcinoma

A
Dysplasia = reversible and has not yet penetrated the BM
Carcinoma = irreversible and has penetrated the BM, histologic findings may be identical
521
Q

Culture negative causes of endocarditis

A

Bartonella, coxiella, mycoplasma, chlamydia and HACEK organisms

522
Q

Earliest fractures seen in osteoporosis

A

Vertebral, this is because its integrity is dependent on cancellous bone, which is first affected in osteoporosis. Cortical bone found in the hip is affected later.

523
Q

3 types of rapidly progressive glomerulonephritis

A

1) Anti-GBM abs
2) Immune complex deposition
3) Pauci-immune

524
Q

Primary cause of morbidity in patients with acute rheumatic fever

A

Pancarditis

525
Q

Cause of nipple retraction in patients with breast cancer

A

Infiltration of the suspensory ligaments of Cooper

526
Q

A 5 year old presents with pruritus and erythematous weeping/crusted papules and plaques whenever he eats certain foods. What is likely causing his condition

A

Atopic dermatitis.

527
Q

NK cell identifiers and function

A

Identified by CD16 and/or CD56.

Kill virally infected or neoplastic cells with decreased MHC I expression.

528
Q

Immune cells involved in superficial vs. hematogenous candida infections

A

Superficial: T-cells
Hematogenous: neutrophils

529
Q

Where are enterococci found as normal flora

A

Urogenital tract and GI tract

530
Q

Metyrapone stimulation test

A

Metyrapone inhibits 11beta-hydroxylase, preventing formation of cortisol. Normally this results in decreased negative feedback to the pituitary and hypothalamus, increased secretion of CRH and ACTH and increased production of 11-deoxycortisol. 11-deoxycortisol is converted to 17-hydroxycorticosteroids that are excreted in the urine and pituitary response to decreased negative feedback can be assessed. If 17-hydroxycorticosteroid synthesis did not risk appropriately, serum ACTH levels will tell you if this is primary vs. secondary adrenal insufficiency.

531
Q

Repair of pyrimidine dimers from UV light

A

Nucleotide excision repair with nicking endonuclease, DNA polymerase and DNA ligase. Mutations in this system = xeroderma pigmentosum.

532
Q

Repair of spontaneously deaminated DNA bases (T -> U and adenine -> hypoxanthine)

A

Base excision repair with glycosylases.

533
Q

Location of lesion in a patient with complete contralateral sensory loss?

A

Ventral posterior thalamic nucleus (VPL = spinothalamic tract + dorsal column sensation, VPM = trigeminal sensation)

534
Q

Nerve you could stimulate to improve sleep apnea

A

CN IX to push the tongue forward

535
Q

Innervation of the tongue

A

Anterior to the foramen cecum and terminal sulcus:

  • taste = chorda tympani (CN VII)
  • sensation = lingual nerve (CN V3)

Posterior to the foramen cecum and terminal sulcus:
-glossopharyngeal nerve

CN XII provides motor innervation for all muscles except for the palatoglossus (CN X)

536
Q

Most common cause of bacteremia in sickle cell patients

A

1) S. pneumoniae
2) H. influenzae

This is why they should get PCN prophylaxis and PCV13

537
Q

How does C. diphtheria acquire its toxin?

A

Bacteriophage incorporation of the Tox gene into cornyebacterium

538
Q

When do bacillus anthracis and clostridium tetani produce toxins

A

B. anthracis > 37C

C. tetani = only in anaerobic environments

539
Q

Cause of worse symptoms with second Dengue infection

A

Typically due to infection by one of the other 4 serotypes

540
Q

Curling vs. Cushing ulcers

A

Curling: trauma/burns, in proximal duodenum
Cushing: intracranial injury, in esophagus, stomach or duodenum

541
Q

Ethosuxamide mechanism of action

A

Blocks T-type Ca2+ channels

542
Q

Drug of choice in patients with generalized tonic-clonic seizures and associated absence seizures

A

Valproic acid

543
Q

Osteoblastic metastases

A

Prostate
Hodgkins
Small cell lung

544
Q

Skin lesions of primary, secondary and tertiary syphilis

A
Primary = painless chancre that dissolves ~ 1 month
Secondary = condyloma lata (large, gray wart-like growths on genitalia)
Tertiary = gumma (white-gray rubbery lesions that may ulcerate)
545
Q

Stage you see neurosyphilis

A

ANY STAGE!

546
Q

Drainage of the rectum

A
Superior = superior rectal vein -> inferior mesenteric veins (portal circulation)
Middle = middle rectal vein -> internal iliac vein
Inferior = inferior rectal vein -> internal pudendal -> internal iliac veins
547
Q

Brain mass with:
A) pseudo rosettes with GFAP + processes tapering toward blood vessels
B) small round blue cells that occasionally form Homer-Wright rosettes
C) poorly differentiated, pleomorphic astrocytes with marked nuclear atypic, mitosis surrounding an area of necrosis
D) spindle cells with hair-like glial processes associated with micro cysts and eosinophilic wavy fibers

A

A) Ependymoma
B) Medulloblastoma
C) GBM
D) Pilocytic astrocytoma

548
Q

Internal capsule strokes

A

Pure sensorimotor deficits (no aphasia, apraxia, anopsia because there is no cortical involvement)

Note that if it is only the posterior limb, it will be motor + ataxia

If it is only the genu, then you get dysarthria-clumsy hand syndrome

549
Q

Genetic mutation associated with dilated cardiomyopathy

A

Dystrophin and other cytoskeletal proteins

550
Q

Ursodeoxycholic acid mechanism of action

A

Decreases biliary cholesterol content and increases bile acid content to prohibit precipitation of cholesterol stones

551
Q

Pathogenesis of Lupus pancytopenia vs. Lupus nephritis

A
Pancytopenia = type II hypersensitivity
Nephritis = type III hypersensitivity
552
Q

Pathogenesis of hyperestrinism in cirrhotics

A

Increased adrenal production of androstenedione -> aromatization to estrone by aromatase -> eventual conversion to estradiol -> increased hepatic production of SHBG and increased preferential binding of testosterone to SHBG, increasing the ratio of estrogen to free testosterone

553
Q

Mechanism of action of bupropion

A

DAT + NET re-uptake inhibitors. This is why this drug is activating and is first line for patients who do not want sexual dysfunction.

554
Q

Length constant (space constant)

A

How far along an axon an action potential can propagate

555
Q

Spatial vs. temporal summation

A

Spatial: multiple neuronal impulses to dendrites, cell body or axon hillock from several different neurons

Temporal: multiple impulses from the same neuron

556
Q

Weber result in a patient with sensorineural hearing loss

A

Lateralizes to unaffected ear

557
Q

What intracellular organelle is crucial in the expression of antigen-bound MHC II on APCs? Why not in MHC I?

A

Phagolysosome. The MHC II is complexed with an invariant chain in the rER, which gets degraded in the acidic phagolysosome, allowing antigen binding to the MHC II. Once the antigen is bound, MHC II can be expressed on the outer membrane surface.

In MHC I, self antigen, tumor antigen or viral protein is degraded by the proteasome and brought into the rER to bind the antigen site of MHC I. It is then packaged in vesicles and sent to the cell surface.

558
Q

Treatment of enterobius (pinworm) infection

A

Albendazole (pyrantel pamoate if pregnant)

559
Q

Parasites treated with diethylcarbamazine

A

Loa loa and wuchereria bancrofti (lymphatic filariasis)

560
Q

Parasites treated with ivermectin

A

Strongyloides and onchocerca

561
Q

Parasites treated with nifurtimox

A

Trypanosomes

562
Q

Parasites treated with praziquantel

A

Schistosoma, clonorchis and paragonimus

563
Q

Childhood viral illness with:
A) High fever followed by truncal maculopapular rash that spreads to the face and extremities 3-5 days later
B) Maculopapular rash that starts on the face and spreads to the extremities with soft palate Forchheimer spots, posterior auricular and occipital lymphadenopathy
C) Maculopapular rash that starts on the face and spreads to the extremities with buccal Koplik spots

A

A) Roseola
B) Rubella (German measles)
C) Rubeola (measles)

564
Q

How do CCBs and NO work to cause blood vessel relaxation

A

CCBs: block Ca2+ influx necessary for muscle contraction
NO: increase cGMP phosphorylation of myosin light chain kinase, inactivating it AND decreasing intracellular Ca2+

565
Q

When are HIV patients at increased risk for opportunistic histoplasmosis

A

CD4+

566
Q

Differentiating MAC from M. Tb

A

MAC grows optimally at 41C where M. Tb does not. MAC also has more significant involvement of the reticuloendothelial system

567
Q

Drug of choice for beta-blocker overdose

A

Glucagon: it increases cAMP and intracellular Ca2+ levels, stimulating muscle contraction and improving HR and BP within minutes.

568
Q

Antibodies that are specific for SLE

A

Anti-dsDNA and Anti-Smith

569
Q

Drug to avoid in patients who are positive for the HLA-B*57:01 allele

A

Abacavir, it can cause a type IV hypersensitivity reaction characterized by fever, malaise, GI symptoms and delayed rash.

570
Q

Drug you can inject locally for patients on NE drip who experience induration and pallor of tissue around NE infusion site.

A

Phentolamine (alpha-1 blocker)

571
Q

Pathogenesis of keloid formation

A

Fibroblasts in hypertrophic scars tend to have increased TGF-beta expression.

572
Q

UMN and LMN signs of syringomyelia

A

LE: spasticity and weakness in the lower extremities and loss of vibration and proprioception in the feet
UE: weakness, hyporeflexia, loss of temperature/touch sensation in bilateral upper extremities

573
Q

Why does diffuse gastric adenocarcinoma diffusely infiltrate

A

Loss of E-cadhedrin

574
Q

Menetrier disease

A

Gastric rugal hypertrophy with atrophy of parietal cells and decreased acid secretion

575
Q

Consequence of
A) AICA occlusion
B) PICA occlusion

A

A) lateral inferior pontine syndrome: loss of contralateral pain and temperature and ipsilateral CN V (facial sensation and mastication) and VIII (vertigo) loss, Horner’s syndrome and cerebellar symptoms

B) lateral medullary syndrome (Wallenberg): contralateral loss of pain and temperature, ipsilateral loss of CN V, VIII, IX, X and XI + Horner’s syndomre

576
Q

Drugs that may precipitate a spike in plasma renin activity`

A

ACE-I, ARBs (loss of feedback) and HCTZ (hypovolemia)

577
Q

alpha-1 second messenger system

A

IP3 + DAG

578
Q

beta adrenergic second messenger system

A

cAMP

579
Q

M1 + M3 second messenger system

A

IP3 + DAG

580
Q

M2 second messenger system

A

Gi -> decrease cAMP

581
Q

Microsporum fungi cause what conditions?

A

Tinea capitis and corporis

582
Q
The following drugs are chelators for...
A) Penicillamine
B) Dimercaprol
C) Lactulose
D) Deferoxamine
E) EDTA
F) Dexrazoxane
A
A) Cu
B) Pb
C) NH3 -> NH4+ (not reabsorbable in gut)
D) Fe
E) Pb
F) Fe (used to limit CM-producing radicals in anthracycline use)
583
Q

McCune-Albright pathophysiology

A

Mosaic GNAS mutation -> constitutive Gs activation

  • Cafe-au-lait spots due to Gs activation in melanocytes
  • Precocious puberty due to autonomous endocrine activation
  • Fibrous dysplasia due to fibroblast proliferation and IL-6 induced osteoclast activation
584
Q

Brodmann areas of the motor and pre-motor cortex

A

Motor: 4

Pre-motor: 6

585
Q

Why does V. cholera require such a large infectious dose to infect a patient?

A

It is extremely acid sensitive

586
Q

Supplement given to patients with measles

A

Vitamin A stores are depleted in measles infection. Supplementation limits keratitis/corneal ulceration, PNA, encephalitis, improves recovery time and shortens length of hospital stays.

587
Q

Vasculitis demonstrating:
A) transmural inflammation of the arterial wall + fibrinoid necrosis in all vessels except the lung, associated with HBV
B) small vessel vasculitis associated with antibiotic use
C) small vessel vasculitis with granulomas, asthma and eosinophilia
D) inflammatory thrombosis and ischemia in small to medium arteries leading to gangrene in worst cases

A

A) Polyarteritis nodosa
B) Microscopic polyangiitis
C) Churg-Strauss
D) Thromboangiitis obliterans

588
Q

Cells that proliferate in gliosis

A

Astrocytes (not microglia)

589
Q

Diabetic CN III palsy vs. compression CN III palsy

A
Diabetic = ischemia internal 1st -> down and out gaze 1st
Compression = ischemia externally 1st -> blown pupil 1st because parasympathetic fibers run external to CN III fibers
590
Q

Stewart-Treves syndrome

A

Angiosarcoma after lymph node dissection and chronic lymphedema

591
Q

Drugs that act as selective coronary vasodilators

A

Adenosine

Dipyramidole

592
Q

How does lipid A induce shock in patients infected by gram-negative organisms?

A

It activates macrophages and granulocytes leading to increased synthesis of pyrogens (IL-1 and prostaglandins) and inflammatory mediators (TNF-alpha + interferon-gamma)

593
Q

Mechanism of
A) senile presbyopia
B) poor far-sightedness that improves with age

A

A) the lens hardens and contraction of the ciliary muscle and relaxation of the zonular fibers no longer causes thickening of the lens to project the closer image on the macula.

B) in patients with myopia (increased eye length) the image projects infront of the retina. As the patient ages, the lens hardens and the image projects further back on the macula and vision improves

594
Q

Etanercept mechanism of action

A

TNF-alpha receptor is linked to the Fc portion of a humanized IgG molecule and acts as a decoy receptor.

595
Q

Why are patients with Crohn’s disease at higher risk for oxalate stones?

A

Malabsorption of fat increases fat binding to Ca2+. This decreases the Ca2+ binding of oxalate to be excreted in feces and increases absorption of oxalate, which accumulates in the nephron to form stones.

596
Q

Macrocyclic antibiotic that inhibits the sigma subunit of bacterial RNA polymerase

A

Fidoxamicin, this drug has a significantly lower recurrence rate of infection in patients with C. difficile because it has less of an effect on normal flora than metronidazole and vancomycin.

597
Q

Drug given during pregnancy that results in a gray baby

A

Chloramphenicol

598
Q

Blood circulation in the kidney

A

Interlobar -> arcuate -> radial -> afferent -> glomerulus -> efferent -> peritubular capillaries -> interlobar veins

599
Q

Why does GFR decrease over time after efferent arteriole vasoconstriction?

A

The slow flow rate results in accumulation of non-filterable plasma proteins and increased oncotic pressure that eventually overcomes the increased hydrostatic pressure produced by efferent arteriolar vasoconstriction

600
Q

Why does LMWH have greater activity against factor Xa?

A

It does not have a pentasaccharide chain long enough to bind both anti-thrombin and thrombin (like heparin does, which allows it to inactivate both and have greater efficacy at inhibition of thrombin).

601
Q

Fondaparinux

A

Pentasaccharide factor Xa inhibitor

602
Q

Histologic features of celiac disease

A

Intestinal crypt hyperplasia, increased lymphocytes and blunted villi

603
Q

Streptokinase mechanism of action

A

Complexes with plasminogen to cleave it to plasmin and split fibrin clots. It also destroys factors V and VII.

604
Q

Drug used for a-fib in a patient who demonstrates QRS widening during and immediately after exercise

A

Class IC anti-arrhythmias like flecainide and propafenone dissociated slowly from fast Na+ channels. This results in use dependent and exaggerated effects when the heart is firing at faster rates. This is an abnormal finding after exercise because increased conduction usually results in a narrower QRS.

Note that class III anti-arrhythmias that block the K+ channels demonstrate reverse use-dependence with increased effects at lower heart rates.

605
Q

When does the VSD murmur typically arise

A

Days 4-10 when the pulmonary circuit resistance decreases, resulting in increased left to right shunting of blood.

606
Q

Janeway lesion vs. Osler node pathogenesis

A

Janeway = septic emboli

Osler nodes = immune complexes

607
Q

Cytokines responsible for SIRS

A

TNF-alpha
IL-1
IL-6

608
Q

Important risk factors for esophageal squamous cell carcinoma in Asia

A

Chewing betel nut and eating smoked meats

609
Q

Intracranial germ cell tumor location that may cause upward gaze palsy and

A

Pineal gland, this is due to its proximal location to the superior colliculus.

610
Q

What is the biggest antibody difference seen in patients receiving the Sabin vs. the Salk polio vaccine?

A

Sabin: antigen exposure -> B-cells in Peyer’s patches activated -> B-cell migration to just beneath lamina propria -> differentiate into plasma cells and secrete IgA dimers linked by a J-chain -> dimers bind to pIgR (polymetric Ig receptor) on the basolateral surface of enterocytes and undergo transcytosis through the enterocyte and into the gut lumen.

Salk: no direct antigen exposure in the gut and more of an IgM and IgG response

611
Q

Osteogenesis imperfecta inheritance

A

AD

612
Q

Causes of acute acalculous cholecystitis

A

Stress (burns, sepsis, trauma, immunosuppression)

613
Q

Why do patients with fat emboli get petechiae?

A

Platelets covert the fat embolus with resulting thrombocytopenia

614
Q

Stain that makes fat globules black

A

Osmium tetroxide

615
Q

Genetic mutation that causes ALS

A

SOD1 -> defective Cu-Zn superoxide dismutase

616
Q

Common secondary malignancies that occur in familial retinoblastoma

A

Osteosarcoma. It is also associated with mutation of the Rb tumor suppressor gene (prevents G1 -> S transition) on chromosome 13.

617
Q

Different types of muscle fibers

A

I: slow, red, oxidative phosphorylation w/lots of myoglobin and mitochondria

IIa: fast, oxidative

IIb: fast, glycogen only

618
Q

Most important prognostic indicator in PSGN?

A

Age

619
Q

Treatment of PSGN?

A

Loop diuretics and vasodilators to treat symptoms, corticosteroids are not effective

620
Q

Why do patients with Dandy-Walker malformation present with progressive skull enlargement?

A

Malformation of the cerebellum and 4th ventricle can result in obstructive hydrocephalus due to agenesis of the foramena of Luschka and Magendie

621
Q

Risk of cancer in a patient with duodenal ulcer + H. pylori infection

A

Minimal. H. pylori has associated increased cancer risk with gastric ulcers.

622
Q

A patient develops a pelvic hematoma after low transverse cesarian section is performed. What happened?

A

Transection of the interior epigastric vessels, hematoma occurs at this location because it is below the arcuate line and there is no posterior rectus sheath in this location.

623
Q
Effect of inhaled anesthetics on:
A) Cardiovascular system
B) CNS
C) Respiratory system
D) Kidneys
E) Liver
A

A) myocardial depression -> decreased CO + low BP
B) increased GABA transmission and cerebral blood flow
C) respiratory depression + decreased mucociliary clearance
D) decreased GFR, renal plasma flow and increased renal vascular resistance
E) decreased hepatic flow

624
Q

Expanding aneurisms of what arteries can result in CN III palsy?

A

It courses between the posterior cerebral and superior cerebellar arteries as it leaves the midbrain and is susceptible to aneurisms in either vessel.

625
Q

What does TGF-beta do?

A

Cell cycle arrest, angiogenesis (tumor cells must first become resistant to the cell cycle arrest component before they can benefit from angiogenic properties) and fibroblast stimulation to lay down extra-cellular matrix proteins.

626
Q

ANP and BNP signal transmission

A

G-protein activation of guanylyl cyclase

627
Q

Why patients get diarrhea with a gastrinoma

A

Excess acid inactivates pancreatic enzymes

628
Q

RAAS components elevated when using ARBs

A

Drop in blood pressure = increase in renin, ATI and ATII. Aldosterone lowers due to decreased ATII stimulation of adrenals and bradykinin remains the same because ACE is not inhibited.

629
Q

Nerves involved in the biceps, brachioradialis and triceps reflexes

A

Biceps + brachioradialis = C5-C6

Triceps = C7-C8

630
Q

How do you assess lymphocytes for monoclonality when suspicious of lymphoma?

A

T-cells: identical genes coding for V region

B-cells: identical Ig variable regions

631
Q

Gardos channel blocker function

A

The Gardos channel is a Ca-dependent potassium channel that, when blocked, results in K+ and H2O accumulation within RBCs. This prevents RBC dehydration and limits RBC sickling in patients with sickle cell disease.

632
Q

Treatment of chloroquine resistant malaria?

A

Mefloquine, quinine

Add primaquine to cover hypnozoites if P. vivax or ovale (unless G6PD deficient)

633
Q

Pathogenesis of digital clubbing

A

Pulmonary disease results in poor platelet fragmentation. Platelet clumps and megakaryocytes deposit in peripheral tissue like the fingers and release PDGF, VEGF and PGE2 which causes local growth and fingernail clubbing.

634
Q

Only type of ribosomal rRNA not transcribed in the nucleolus

A

5S rRNA.

635
Q

Most common mutation of Duchenne muscular dystrophy

A

Deletion of the dystrophin gene at p21 on the X chromosome resulting in a frameshift mutation. Non-frameshift deletions result in Becker muscular dystrophy

636
Q

Histologic markers of a leukemoid reaction (WB > 50k) vs. leukemia

A

Leukemioid reactions have normal to high levels of leukocyte alkaline phosphatase, Dohle bodies (oval basophilic inclusions within mature PMNs), toxic granules and cytoplasmic vacuoles.

637
Q

Vessels that most often develop atherosclerotic plaques

A

1) Abdominal aorta
2) Popliteal artery
3) Coronary arteries
4) ICA
5) Circle of Willis

638
Q

How to make the diagnosis of malabsorption?

A

1) Screen with Sudan III stool stain for fecal fat

2) Follow up quantification of fecal fat content, > 7g/day = malabsorption

639
Q

Best correlation with duration and severity of chronic bronchitis?

A

Reid index > 40%

640
Q

Pathophysiology of dermal atrophy with extended use of corticosteroids

A

Inhibition of fibroblasts resulting in reduced laying down of extracellular collagen and glycosaminoglycans.

641
Q

Cytoplasmic inclusions of tau protein

A

Pick bodies, contrast to Tau proteins that make up neurofibrillary tangles.

642
Q

Pathogenesis of toxic shock syndrome

A

TSST-1, enterotoxin and exfoliative toxin cause tight binding of T-cells to APCs and massive release of cytokines. IL-2 release from T-cells and TNF-alpha + IL-1 from macrophages results in shock.

643
Q

How to locate the appendix

A

Follow the bands of tenia coli to where the converge at the base of the appendix near the cecum.

644
Q

How to choose your anti-emetic

A

GI disturbance from infection, chemo or anesthesia: results in increased 5-HT release from mucosa and nausea, treat with 5-HT3 antagonist like ondansetron

Central nausea (migraine): DA antagonists like promethazine and metoclopramide

Motion sickness: first generation H1 blockers (diphenhydramine, meclizine) and muscarinic antagonists (scopolamine)

645
Q

Pathophysiology of pancreatitis due to alcohol use?

A

It causes protein rich fluid secretion that can clog ducts, it is directly toxic to acinar cells and causes spasm of the sphincter of Oddi.

646
Q

Genital ulcers with ragged borders and gram stain that shows gram negative rods in a school of fish chain

A

H. ducreyi

647
Q

Characteristics of inhaled anesthetic with rapid rate of onset

A

Low tissue solubility (low arteriovenous concentration gradient)

648
Q

Lentiform nucleus

A

Putamen + globus pallidus. This is where Cu deposits in Wilson’s disease

649
Q

NSAID that will not cause defects in platelet aggregation

A

Aspirin irreversibly inhibits platelet COX-1, resulting in impaired TxA2 synthesis for the entire life of the platelet (3-7 days)

COX 1 inhibitors reversibly inhibit TxA2 production for 3-4 days.

COX 2 inhibitors have no effect production of TxA2 and platelet aggregation is preserved, however, note that PGI2 may be decreased and result in hypercoaguability.

650
Q

Why might light touch and pressure sensation be preserved in a patient when pain and temperature sensation is lost?

A

Light touch and pressure sensory fibers lie in the anterior spinothalamic tract, as opposed to the pain and temperature fibers that run in the lateral spinothalamic tract.

651
Q

Weird medications that can cause osteoporosis

A

Anticonvulsants (phenobarbital, phenytoin and carbamazepine) increase vitamin D catabolism.

Glucocorticoids, unfractioned heparin and thiozolidinediones reduce bone formation.

652
Q

How can you use the transepithelial potential difference in the nasal mucosa to diagnose someone with cystic fibrosis?

A

In mucosa, the CFTR channel normally functions to push Cl- out and inhibit Na+ entry via ENaC to reduces mucus viscosity. When the channel doesn’t work, mucosal cells have a higher intracellular Na+ content and will have a higher transepithelial potential difference.

This is in contrast to the function of CFTR in sweat glands, which is to absorb lumenal Cl- and increase Na+ absorption through the ENaC. Defective CFTR results in inability to absorb NaCl and salty skin.

653
Q

A patient presents with seizures in the setting of cutaneous angiofibromas, renal angiomyolipomas, cardiac rhabdomyomas and cortical/subependymal hamartomas.

A

Tuberous sclerosis and AD condition

654
Q

How do cholinergic drugs cause vasodilation?

A

Although they do not innervate blood vessels, they do stimulate NO release from endothelial cells, resulting in vasodilation.

655
Q

Other than improvement with movement, what helps differentiate Lambert-Eaton from Myasthenia Gravis?

A

Lambert Eaton may have associated hyporeflexia and autonomic symptoms

656
Q

Labs included in MELDS score for liver failure

A

Albumin, PT and bili. These best demonstrate liver function.

657
Q

What makes sputum green?

A

Myeloperoxidase released from PMN azurophilic granules

658
Q

What are interferon gamma release assays testing

A

Th1 IFN-gamma release upon exposure to M. Tb antigen.

659
Q

Regions of the brain stimulated for Parkinson’s patients to improve movement

A

Sub thalamic nucleus (activates GPi) and GPi (inhibits thalamus)

660
Q

Pump inhibited by PPIs

A

H/K ATPase

661
Q

Pupillary light reflex pathway

A

Light -> CN II -> ipsilateral pre-tectal nucleus -> bilateral Edinger-Westphal nuclei -> bilateral ciliary ganglion -> bilateral pupillary constrictors

662
Q

What genetic mutation in NSCLC is similar to the BCR:ABL mutation seen in CML?

A

EML4:ALK mutation is often seen in young non-smokers with adenocarcinoma and no other mutations in the EGFR or K-ras genes.

663
Q

Where does the thoracic duct drain into?

A

The left subclavian vein at its junction with the IJ

664
Q

Veins that join to form the portal vein

A

Splenic and superior mesenteric veins

665
Q

Treatment of trigeminal neuralgia

A

Carbamazepine is effective in 80% of patients as it slows Na+ channel recovery and reduces frequency of Na+ channel firing

666
Q

Why is storage and later consumption of soybeans, corn and peanuts associated with liver cancer?

A

They allow for growth of aflatoxin-producing aspergillus. Aflatoxins induce mutations in p53 that lead to unregulated cellular proliferation and cancer.

667
Q

How do CCBs cause AV node block?

A

They block L-type Ca2+ channels, which decreases the rate of phase 4 depolarization and conduction velocity in both SA and AV nodes.

668
Q

3 meds used for MRSA and their side effects

A

1) Vancomycin: binds D-ala-D-ala and blocks glycopeptide polymerization, red man syndrome + nephrotoxicity
2) Daptomycin: depolarizes cell membrane by creating ion channels, myopathy, CPK elevation + inactivation by pulmonary surfactant
3) Linezolid: inhibits 50S by binding the 23S subunit and prevents formation of 70S ribosome, thrombocytopenia, serotonin syndrome and optic neuritis

669
Q

Immature granulocytes that can be seen in the blood of a person having a leukemoid reaction?

A

Myelocytes and metamyelocytes. Make sure the LAP is normal or elevated before chalking it up to a leukemoid reaction. Also, CML would show increased eosinophils and basophils too.

670
Q

Blast cell level in ALL and AML

A

Typically > 20%

671
Q

Micturition reflex

A

Sacral micturition center: S2-S4 parasympathetics, causes bladder contraction

Pontine micturition center: coordinates relaxation of external urethral sphincter with bladder contraction

Cerebral cortex: paraventricular neurons inhibit the sacral micturition center (injured in NPH)

672
Q

Why are patients in the maintenance stage of ATN at risk for a gap metabolic acidosis?

A

Retention of H+ and anions like SO4-, PO4- and urate

673
Q

Total resistance in a group of vessels arranged in parallel?

A

TPR = 1 / (1/R1 + 1/R2 + 1/R3 + 1/R4)

674
Q

Where do the leads go on a biventricular pacemaker

A

1) Right atrium
2) Right ventricle
3) Coronary sinus in the atrioventricular groove

675
Q

Mechanism of HUS

A

Veratoxin damages glomerular capillaries and blood vessels, exposing collagen and causing widespread platelet activation, thrombus formation and MAHA.

676
Q

Nerve injury in CNS vs. PNS

A

In the PNS, a transected nerve results in Schwann cell activation and release of mediators to recruit macrophages to phagocytose the myelin sheath. They also release trophic factor that stimulates formation of the growth cone from the transected portion of the proximal neuron.

In the CNS, oligodendrocytes are inactive and do not participate in phagocytosis. Monocyte recruitment is slow due to the BBB. The remaining myelin has inhibitory factors that prevent neuronal regeneration. Also, astrocytes come and form the glial scar, which also secrete inhibitory molecules.

677
Q

Ranke complex

A

Healed scar after primary Tb infection, not associated with reactivation